Re: Quantum Immortality and Information Flow

22 views
Skip to first unread message

Bruno Marchal

unread,
Nov 16, 2005, 10:35:30 AM11/16/05
to Fabric-o...@yahoogroups.com, Everything-List List
Le 15-nov.-05, à 10:56, Brian Scurfield a écrit :

> --- In Fabric-o...@yahoogroups.com, Bruno Marchal <marchal@u...>
> wrote:
>
>>> It has often been pointed out on this list that universes are those
>>> parts of the multiverse down which information flows. So Harry
>>> Potter "universes" are not in fact universes.
>>
>> What do you mean by "parts of the multiverse down which information
>> flows"?
>
> OK, let's start with information. I have in mind David's qualitative
> definition here:
>
> http://xxx.lanl.gov/abs/quant-ph/0104033
>
> To quote:
>
> 1. A physical system S *contains information* about a parameter b if …
> the probability of some outcome of some measurement on S depends on b.
>
> 2. A physical system S *contains no information* about b if … there
> exists a complete description of S that is independent on b.
>
> What is meant by information flow is explained here:
>
> http://groups.yahoo.com/group/Fabric-of-Reality/message/9247
>
> Basically, information flows when the output depends in some way on
> the input.
>
> As David has shown, the structure of the multiverse is determined by
> information flow. A universe is a part of the multiverse where
> information flows freely.

OK. I guess you know that this is not a standard use of the term
"information", like the one of Shannon, where the bits measure some
degree of surprise and unpredictibility (and this can been make precise
with notion of Kolmogorov/Chaitin/Solovay notion of information).
David Deutsch's notion of information can be related to some logician's
attempt to define a sort of qualitative information, and as such it is
quite interesting, but also natural, to find it in an attempt to
retrieve classical computational histories from the quantum theory.
Now you should keep in mind that David does postulate the existence of
some continuum causal structure, and, as you know, I am doubtful this
can make sense once we accept the computationalist hypothesis (cf
Maudlin's Olympia, my work, etc.). Actually I think David Hume makes
already important steps in that direction.
Just to illustrate: imagine the case of iterated self-duplication with
reconstitutions in Sidney and Beijing: S and B. From the point of view
of the "average candidate" normal histories, in the form of sequences
of P and S, will be highly unpredictable, but in all case there will
be no causal relations between the events "I feel myself to be in S" or
"I feel myself to be in P".
Exceptional histories, like the one in which the sequence gives the
binary digits of the number PI, play the role, curiously enough, of the
Harry Potter histories, in that case.
So the two notion of information are quite complementary. It is almost
like one grows up when the other diminishes.


>
>> Harry Potter universes are just improbable, and information grows to
>> much there.
>
> Things spontaneously organise themselves in an HPU, but the output
> does not depend on the input; there is no information flow in the
> sense described above.

So I agree. And of course I was considering a Shannon form of
information.

> When Harry Potter does a trick, in almost all
> universes the trick does not work. But one cannot say the trick
> succeeded in those universes that do become HPU's because one can't
> single out beforehand those exceptionally rare universes that will
> become HPU's.

I agree.

>
>> It is almost like they got singularities in the amount of
>> information. But death, well, really it is an open problem, because
>> you must take into account the normal (statistical, based on the
>> measure on the "observer/moment/states/worlds...") possible
>> histories just going locally through exceptional states,
>
> Meaningful histories must have a flow of information right?

I don't think so. I would say that meaningful histories are the
relatively consistent (non contradictory) one---which will get some
right measure assuming comp, and the correctness of my derivation, to
be sure. Those meaningful histories will appear, from the observer
first person point of view be related to some local flux of information
in David's sense. But I don't know how to take for granted those
meaningful histories at the start (unless, like David, you already
postulate some "prior programs" as an explanation of the appearance of
the universe. But then again you will be confronted with the comp
version of the Mind/Body, or 1-person/3-person relations again.

> In some
> histories, the Universal Dovetailer has outputted a series of random
> numbers that just happen to be indistinguishable from your life
> history up to today. In these histories, your past state does not
> determine your present state (although they are in every way
> indistinguishable from histories where your past state really does
> determine your present state).

I would say you belong to all histories. But with comp you are so
constituted that only the consistent one will make sense and will be
stable enough.

> Almost all of these histories will turn
> to junk at the very next moment. But some won't.

Yes but here you touch the real difficulty which could one day kill the
comp hyp. The difficulty is that "junk" can be consistent. Harry Potter
universe are consistent. White Rabbits are consistent. We can only
thrown them away once we have shown they have low relative measure.
Deutsch's justification is correct but incomplete, I think. It is
complete only by postulating the existence of a causal realm, or of a
special prior universal program a-la-Schmidhuber. I have already
criticize such move once we assume comp keeping in mind the distinction
between the points of view.


> Let's suppose that a
> huge meteorite is about to crash into your house.


Let us suppose .... (thanks!!!)

> In most histories
> you will be dead and vapourised.


Just to be clear, you and all other people (other than me) will seen me
dead (and vaporized if you insist ;).
I will most probably not feel myself dead or vaporized (obvious with
comp).

> But in some of the random histories
> that give the illusion of order and information flow you survive
> because the next state just happens to correspond to you sitting alive
> - looking very black and charred - in a giant crater.
>
> Can you really be said to be alive post-impact if these random
> histories are all that is left?
>
> I guess you will answer that there will still be histories post-impact
> where your next state really does depend on your previous state even
> if those histories were random (in just the right way) during impact.
> Correctomundo?


Quite correct. Why do you ask if you know the answer? You see, a
machine cannot distinguish random stories (Shannon information high,
Deutsch information low) from complex stories (Shannon information low
or much less high, and Deutsch information high). In the normal
situations, we can indeed survive only in those where Deutsch
information is high, but "near death" we will survive in the "most"
normal relatively to the normal histories which gave rise to us, and
immediately "after" we will proceed in those new relatively normal
extensions.
Deutsch paper is very interesting and I agree with many things there,
but I would not take notion like continuum and causality as granted. I
would say those things, like the "collapse of the wave", emerges from
the relatively consistent observations available to us.

In "the structure of the multiverse" deutsch write:

<<This does not imply that such a subsystem constitutes half the region
of the multiverse in which R exists. Proportions in the latter sense –
which formally play the role of probabilities under some circumstances,
as shown in Deutsch (1999) – are determined by the Heisenberg state as
well as the observables, and do not concern us here because the present
discussion is not quantitative.>>

My general problem here would be to find a way to relate more clearly
the quantitative shannon form of information and its probability link,
with the Deutsch information notion.
Some logician like Abramsky (is he in Oxford?) seems to have make
attempts in similar direction. Keith Devlin wrote also a book on
"non-shannon" like information.

Another direction would consist in deepening the interview with the
universal (lobian) machine. Each theaetical knowledge notion should
lead to an information structure. I cannot explain this without digging
more on the modal logic of self-reference, and the theaetical
transformations. So ask me on the everything list if you want me to
make clearer that last paragraph. It could interest also the FOR
people, because I do think that the difference between "p &
provable(p)" with "consistent(p) & provable(p)" is akin to the
"correction", proposed by Popper/Deutsch and Rafe Champion here, of
the platonist "true opinion" notion of knowledge (= first theatetical
trick) which leads to a more conjectural from of knowledge. By
incompleteness "consistent(p)" is necessarily conjectural.

..must go now. Hope I'm clear enough.

Bruno


http://iridia.ulb.ac.be/~marchal/


James N Rose

unread,
Nov 16, 2005, 11:18:24 PM11/16/05
to Everything-List List
An open hypothesis to list members:

"Conservation" as a 'fundamental rule of condition'
is incompatible and antithetical with any notions
of "many worlds".

Either explicitly excludes and precludes the other;
can't have both and retain a consistent existentialism.

J Rose

Russell Standish

unread,
Nov 17, 2005, 12:27:13 AM11/17/05
to James N Rose, Everything-List List
I don't see why. Conservation of information is a fundamental property
of the Multiverse, and is directly equivalent to the law of unitary
evolution in quantum mechanics.

If you are talking about conservation of energy, are you aware that
the total energy content of the universe is zero? All of mass-energy
is balanced by the negative potential energy of gravitational
attraction. Multiplying zero energy universes into a multiverse still
conserves energy. Ditto with momentum - the total momentum of the
universe is zero.

Cheers

--
*PS: A number of people ask me about the attachment to my email, which
is of type "application/pgp-signature". Don't worry, it is not a
virus. It is an electronic signature, that may be used to verify this
email came from me if you have PGP or GPG installed. Otherwise, you
may safely ignore this attachment.

----------------------------------------------------------------------------
A/Prof Russell Standish Phone 8308 3119 (mobile)
Mathematics 0425 253119 (")
UNSW SYDNEY 2052 R.Sta...@unsw.edu.au
Australia http://parallel.hpc.unsw.edu.au/rks
International prefix +612, Interstate prefix 02
----------------------------------------------------------------------------

rmiller

unread,
Nov 17, 2005, 1:48:57 AM11/17/05
to James N Rose, Everything-List List, GARY...@aol.com

I haven't kept up with this thread or that idea, but there is no logical
reason that a particular attribute such as "conservation" should be
universal across a many-world manifold. First of all, "conservation" is
ill-defined, but if precisely defined assumes a standard, which implies a
teleological approach. And that is one step away from
scholasticism. Before you know it, you're quoting Plato. Mathematically,
conservation could be defined in terms of least-distance between points,
but if the individual worlds are constructed with their own unique
space-time topology (sort of by definition--otherwise each world would be
the same as the next one) then the term "conservation" would apply only
locally. So, strike two. In fact, one could describe each world as a
unique slice intersecting and *forming* the surface of the many-world
manifold---and each slice could be characterized by its own unique
matrix. Postulating the individual world matrix as a set of elements and
interactions between elements, one could arrive at an "ideal" (Plato
again!) in which each individual world is confined to a minimum number of
elements/interactions. Fine. But it would result in each world being
congruent (homologous) to every other world. The result would be no
difference between worlds, but there is not a shred of evidence that the
configuration works that way at all levels. For example, you coffee may
have cooled according to the observations setting forth the laws of
thermodynamics---and thus predictable, but you sir, probably drove your
automobile in a very inefficient manner today, going places that you
shouldn't have gone (you didn't know the queue would be so long, or the
store would be closed, etc). Now, if you had known that the store would be
closed, etc, you would have been a little more efficient, but that would
require a prescience that you presumably don't have. Maybe that's why, we
can never precisely predict where the electron will "be", because to do so
would identify it's "proper" place---and from there we could then define
it's ideal position. That we cannot (as yet) do that suggests that this
inability to do so is an inherent part of a dynamic system---and is present
within all intersects of the many world manifold.

Short answer: Conservatism is a procedure that produces mental constructs
of what we thing the world is trying to become. It allows us to fit our
observations against the image in our minds, but it has its
limitations. There is no perfect river. Or snowstorm. Or
politician. It's all in our minds.


scerir

unread,
Nov 17, 2005, 2:12:05 AM11/17/05
to James N Rose, Everything-List List
From: "James N Rose"

> "Conservation" as a 'fundamental rule of condition'
> is incompatible and antithetical with any notions
> of "many worlds".

Are conservation principles only defined in
closed systems? Is a 'world' a closed system?

There is, i.e., a no-deleting theorem (similar
to the no-cloning theorem) saying that given
2 qubits in unknown and equal states, one
cannot take one of them to a fixed state,
keeping the other in the original state.
In other terms |psi>|psi> --> |psi>|0>
is forbidden. But, of course, it is possible
to delete a quantum state by trowing it out
of that system, out of that world.

s.

Stephen Paul King

unread,
Nov 17, 2005, 10:07:56 PM11/17/05
to everyth...@eskimo.com
Hi James and Russell,

Could a middle ground be found in the notion that "something" is a
differentiated piece of Nothing, where Everything (1st person notion) and
Nothing (3rd person notion) are one and the same? Violations of the notion
of conservation only seem to obtain when we conflate the 1st and 3rd person
viewpoints.

The problem that I see with existentialism is that it tacitly assumes an
unattainable 3rd person upon which to base its notion of existence. Why do
we need to assume more than "Existence exists" (in an active and not passive
sense)?

Onward!

Stephen

Russell Standish

unread,
Nov 17, 2005, 10:51:26 PM11/17/05
to Stephen Paul King, everyth...@eskimo.com
On Thu, Nov 17, 2005 at 10:00:04PM -0500, Stephen Paul King wrote:
> Hi James and Russell,
>
> Could a middle ground be found in the notion that "something" is a
> differentiated piece of Nothing, where Everything (1st person notion) and
> Nothing (3rd person notion) are one and the same?

Intriguing. Why do you say Everything is 1st person and Nothing is 3rd person?

> Violations of the notion
> of conservation only seem to obtain when we conflate the 1st and 3rd person
> viewpoints.
>

This seems likely.

Stephen Paul King

unread,
Nov 17, 2005, 11:28:07 PM11/17/05
to everyth...@eskimo.com
Hi Russell,

It seems logical. The Notion of "Everything" is 1st person in the sense
that one, any one, can find itself within it. Nothing, on the other hand,
only makes sense as seen from some external vantage point, hence it is 3rd
person. This is probably naive, but it makes the whole structure "hang
together". For example, the idea of conservation, ala "you can't get
something from nothing", is a truism for the simple reason that it is not
consistent to have some concept that is definite, a "something", to be
dependent on a Nothing. The notion of something requires an "other" against
which it is distinguished; there is no "other" in Nothingness.

Onward!

Stephen

----- Original Message -----
From: "Russell Standish" <r.sta...@unsw.edu.au>
To: "Stephen Paul King" <step...@charter.net>
Cc: <everyth...@eskimo.com>
Sent: Thursday, November 17, 2005 10:47 PM
Subject: Re: contention: theories are incompatible

George Levy

unread,
Nov 18, 2005, 12:25:45 AM11/18/05
to Everything List

Along the line of Jorge Luis Borges a blackboard covered in chalk contains the library of Babel (everything) but no information. Similarly a white board covered with ink also contains no information.
Interestingly, information is minimized or actually goes to zero when the world is too large as the plenitude, or too small. Information is maximized when the world is neither too large nor too small. We live in a Goldilock world.

George

Bruno Marchal

unread,
Nov 18, 2005, 8:53:19 AM11/18/05
to Stephen Paul King, everyth...@eskimo.com

Le 18-nov.-05, à 05:26, Stephen Paul King a écrit :

> It seems logical. The Notion of "Everything" is 1st person in the
> sense that one, any one, can find itself within it. Nothing, on the
> other hand, only makes sense as seen from some external vantage point,
> hence it is 3rd person.

I can understand why there is no notion of first person nothingness
(and this is the base of the non cul-de-sac appearing with the first
person notion(*).

But for "everything" I think we can have some third person notions.
Typical examples are the complete trace of the running of the UD,
written UD*, or a model of PA, ZF, etc.
The first person notion of everything (the 1-plenitude) is, assuming
comp, so big that it is unnamable by any machine (provably so if the
"person" is some fixed not too complex Lobian machine, like a theorem
prover for PA).

Bruno


(*) For those who remembers the modal introduction: a no-cul-de-sac
multiverse (a multiverse where all observer-moment/world/state are
transitory) verifies the formula []p -> <>p (the so-called deontic
formula d). Note that d is not a theorem of G, but is a theorem of G*.
d is the well known main axiom for the deontic logic of obligation
/permission: indeed a world where d is false is a world where something
is obligatory and not permitted. You can put anyone in jail-cul-de-sac
there!

General question: what do you prefer, as notation (illustrated on the
formula d) :

Box p -> Diamond p
Bp -> Dp
[]p -> <>p ?

Are there people who does not see that
1) whatever the truth value of p, Bp -> Dp is true in all the worlds of
a non-cul-de-sac multiverse.
2) if Bp -> Dp is true in all world of multiverse, whatever the truth
value of p is given in each world, then the multiverse is a
non-cul-de-sac multiverse.

This is easy. If you don't see this, it means you don't remember the
definition of Kripke semantics, or that you don't know classical logic.

Modal logic is really the general theory of Multiverses, and other
multimultiverses, you know. I hardly doubt we will be able to proceed
without getting more familiar with it.
I am actually teaching modal logic and students ask me summary notes. I
am thinking making them in English and posting them to the list. The
post by uv makes me think I should soon or later explain more about
Solovay theorem, which makes the link betwwen the metamathematical
results of Godel, Lob and the G and G* logics discovered by Solovay,
and which are pillar of the interview of the universal machine.

Bruno


http://iridia.ulb.ac.be/~marchal/


Bruno Marchal

unread,
Nov 18, 2005, 10:08:55 AM11/18/05
to Fabric-o...@yahoogroups.com, Everything-List List

Le 17-nov.-05, à 12:40, Brian Scurfield concluded :

>
> OK, I see what you are getting at here, but as you pointed out later
> in your post the problem is that "junk" can be consistent! You want to
> "throw away" the junk by showing it has zero measure without a-priori
> assuming some kind of casual structure based on the continuum.


Exactly. Except I gave an argument that IF the comp hyp. is assumed to
be correct, THEN we *have to* justify that the junk has measure zero
(not just wanting). If we show the junk has a different measure, then
we would refute comp. (Grosso modo: it is the content of the Universal
Dovetailer Argument + movie graph/olympia = first half of my thesis).


> Information flow would then be an emergent property of the consistent
> histories you are left with.

Exactly. Except you can change the "would-be" into a "is". That's the
second half of my thesis, although it is yet an open problem to see if
I got there the right (empirically) information flow. But I get enough
for retrieving a non trivial notion of quantization explaining why the
shape of appearance is necessarily dynamical and non boolean. I hope to
show it being 100% reversible (Newton lesson) and non generally
clonable (Einstein Podolski Rosen lesson).
Well, actually I hope it will gives the qubits.
I am not contesting the Everett-Hartle-... Deutsch-Zurek explanation of
how bits come from qubits. Just saying comp gives a path from bits to
qubits too. A double path.
It is the incompleteness phenomenon(*) which makes that path double,
i.e. separated into a communicable part and a non communicable part
explaining simultaneously quanta and qualia (I would argue).

Bruno

(*) captured by the set difference between the modal logics G* and G,
as I try to explain on the everything-list.


http://iridia.ulb.ac.be/~marchal/


Stephen Paul King

unread,
Nov 18, 2005, 2:43:36 PM11/18/05
to Fabric-o...@yahoogroups.com, everyth...@eskimo.com
Dear Bruno,

Are you claiming that the communicable part is to the non-communicable
part as the classical is to the quantum? The Non-cloning aspect of QM and
the copyability of the classical seems to be implied. Is this intentional?

Onward!

Stephen

----- Original Message -----
From: "Bruno Marchal" <mar...@ulb.ac.be>
To: <Fabric-o...@yahoogroups.com>
Cc: "Everything-List List" <everyth...@eskimo.com>

Sent: Friday, November 18, 2005 10:03 AM
Subject: Re: Quantum Immortality and Information Flow


snip

Bruno Marchal

unread,
Nov 19, 2005, 10:25:34 AM11/19/05
to Fabric-o...@yahoogroups.com, everyth...@eskimo.com

Le 18-nov.-05, à 20:39, Stephen Paul King a écrit :

> Dear Bruno,
>
> Are you claiming that the communicable part is to the
> non-communicable
> part as the classical is to the quantum?

Oops, no, sorry. My fault. I was trying to be short. You can see Godel,
Lob, Solovay discovery as the discovery that the *classical* logic of
self-reference is (already) divided into two parts, corresponding to
the provable and the unprovable. At their propositional levels,
Solovay, in 1975, showed that the modal logics G and G* capture soundly
and even completely the *provable* part of the logic of self-reference
and the *true* part, respectively. The purely unprovable (but true)
part is then given by G* \ G (the set difference of the theories).
"Provable" means provable by some fixed sufficiently rich machine, or
theory.
Now observation and knowledge are defined in the logics of
self-reference, i.e. by transformation of G and G*, and so are each
multiplied by two. Actually and amazingly for the knower (the first
person) G and G* give the same logic, like if the first person
conflates truth and provability. But for the notion of observation, G
and G* give again different logics, so that the observer can
distinguish communicable observations ("physical facts") and non
communicable observations (sensations, I would argue). But to be
honest, the quanta (or the shadows of the quanta) seems to appear at
the G* level, confirming "quantum physics" is a first person plural
notion, i.e. based on bets made by multiplication/differentiation of
populations of individuals; like in Everett (QM without collapse) where
superpositions are contagious to the observers.
To sum up, the difference provable/unprovable or
communicable/incommunicable is inherited by all the transformations of
G, except the one which gives the stronger notion of the first person.
(I guess this one *is* the solipsist who lives in each of us, the one
who needs some education or encouragement for learning to listen to the
"solipsist" living in the others).

Bruno

http://iridia.ulb.ac.be/~marchal/


Russell Standish

unread,
Nov 19, 2005, 5:22:58 PM11/19/05
to Bruno Marchal, Fabric-o...@yahoogroups.com, everyth...@eskimo.com
On Sat, Nov 19, 2005 at 04:22:58PM +0100, Bruno Marchal wrote:
> Now observation and knowledge are defined in the logics of
> self-reference, i.e. by transformation of G and G*, and so are each
> multiplied by two. Actually and amazingly for the knower (the first
> person) G and G* give the same logic, like if the first person
> conflates truth and provability. But for the notion of observation, G
> and G* give again different logics, so that the observer can
> distinguish communicable observations ("physical facts") and non
> communicable observations (sensations, I would argue).

Are you now saying that your operators

Pp = Bp & -B-p

and

Op = Bp & p & -B-p

correspond to "to observe" (Op being "to validly observe" I
suppose)?. Previously, you would say that Pp is "to bet on p", and Op
"to correctly bet on p", which never really made sense to me. What's
the French word you would use for this - I may know it, or perhaps I
can figure the relevant English term from a dictionary.

Cheers

Bruno Marchal

unread,
Nov 21, 2005, 9:48:56 AM11/21/05
to Fabric-o...@yahoogroups.com, everyth...@eskimo.com

Le 19-nov.-05, à 22:56, Russell Standish a écrit :


> On Sat, Nov 19, 2005 at 04:22:58PM +0100, Bruno Marchal wrote:
>> Now observation and knowledge are defined in the logics of
>> self-reference, i.e. by transformation of G and G*, and so are each
>> multiplied by two. Actually and amazingly for the knower (the first
>> person) G and G* give the same logic, like if the first person
>> conflates truth and provability. But for the notion of observation, G
>> and G* give again different logics, so that the observer can
>> distinguish communicable observations ("physical facts") and non
>> communicable observations (sensations, I would argue).
>
> Are you now saying that your operators
>
> Pp = Bp & -B-p
>
> and
>
> Op = Bp & p & -B-p
>
> correspond to "to observe" (Op being "to validly observe" I
> suppose)?. Previously, you would say that Pp is "to bet on p", and Op
> "to correctly bet on p", which never really made sense to me. What's
> the French word you would use for this - I may know it, or perhaps I
> can figure the relevant English term from a dictionary.

Let me first explain in few words a plausible logician conception of a
"multiverse". I borrow the term "multiverse" from FOR, but I think we
should be neutral about what is really a universe, or a world, or a
state, or an observer-moment: the only thing which matter is that we
have many of them, and that they are related by a relation of
accessibility. So a multiverse is just a set W (of elements called
"worlds") and a binary relation R defined on it. Let us use the letter
a, b, c, d, ... for the worlds. So aRb just means that the world b is
accessible from the world a. You can travel from a to b.
Note that I am not pretending that the "real multiverse" (perhaps the
quantum one) is of that type, but it is good to begin with that
familiar sort of Kripke multiverse, and then to correct it.

Now we assume that all the worlds obey classical logic: if p is true
in world a, and if q is true in world a, then propositional formula
like (p & q), (p -> q) etc. are true at a, and ~p is false at a, etc.
In particular, all classical tautologies are true in all worlds of all
multiverse independently of the assignment of truth value to the
sentence letter p, q, r, etc.

The main idea of Kripke has consisted in saying that the modal formula
Bp (also written []p) is true at world a, if p is true in all the
worlds you can access from a. p is relatively necessary at a.
For example, if the world are countries and if you have to pay taxes in
all countries that you can access from where you are, then taxes are
necessary (relatively to a).

That is, p is "necessary" at world a if p is true for all worlds b such
that aRb. It is intuitively normal: a proposition is necessary for you
if it is true in all world you can access.

Then a proposition is possible at world a if it is not necessary that
~a. So "possible p", written Dp, or <>p, can be seen as an abbreviation
of ~B~p. Note that if Dp is true at a, it means there is an accessible
world (where p is true) from a. In particular, given that the constant
true t is true in all worlds, Dt really means I can access to some
world (I am alive, if you want).

Now, there are relation between the structure of the multiverse, i.e.
the nature of its accessibility relation, and the formula which are
true in each world. It should be easy to guess that if the multiverse
is reflexive (i.e. all worlds are accessible from themselves) then the
formula Bp -> p is true in all the worlds, independently of the truth
value of the sentence letters. Slightly less easy: the reverse is true:
if Bp -> p is true in all worlds, independently of the assignment of
true/false to the sentence letters, then the multiverse is reflexive.
We say that the reflexive multiverse characterizes the formula Bp -> p.
It means the formula remains invariant when we travel in that
multiverse.
It can be shown that the symmetrical multiverse, that is those where
the accessibility relation is symmetric, characterizes the formula p ->
BDp. The transitive multiverse characterizes Bp -> BBp. etc.

Of special interest in this thread are the dead-end world, or
cul-de-sac observer-moment (we have use many name for them). A world a,
in a multiverse W, is said to be a dead end or a cul-de-sac world if,
when you are in a, there is no more world in which you can acceded. So,
in such world no proposition are possible, so whatever proposition p
is, ~Dp is always false. By classical logic B~p is always true. This is
true whatever p is, in particular this is true for its negation ~p. So
in a dead end world, all proposition are necessary and none is
possible. Not a funny place!

Now, when B represents the Godel-Lob provability predicate, i.e. when B
represents provability in or by a "sufficiently rich" formal
system/machine, it can be shown that the "humble multiverse", that is
those where all worlds have access to a dead end world, characterizes
B. In that case Dp = ~B~p = "~p is not provable" = "p is consistent"
(because if you cannot prove ~p, you will not get a contradiction by
adding ~p as axiom, that is you will not prove ~p, that is p is
consistent (with your formal theory or for your machine). So "humble
multiverse" characterizes the formula Dp -> ~BDp, which is the second
incompleteness theorem of Godel: consistent p -> not provable
consistent p. So the humble multiverse characterize the machine's
discourses.

This prevents us of defining "probability(p) = 1" in world a by Bp is
true at a, because if a is a dead end then Bp is true (D~p is false)
although the probabilities are senseless.

Now, as you know, I limit the interview of machine to the correct and
consistent one. This is just a mathematical trick. For those machine Bp
-> Dp is true, but not provable by the machine. So we can define
"probability(p) = 1" by "Bp & Dp". It means that we define the
probability one of a proposition p, by p is true in all accessible
worlds and there is (at least one) accessible world. The incompleteness
forces us to put explicitly the consistency as a requirement. It
corresponds to the correct bets, or to the observation of laws
(invariant truth of the multiverse).

Actually, adding that Dp to Bp is so much constraining that we loose
the Kripke multiverse structure in the process, but we get instead a
more interesting proximity relation, and ultimately we get (translating
the comp hyp itself in the language of the machine) an orthogonality
structure on the worlds of the multiverse, making it looking like the
quantum multiverse inferred by the observing physicists.

Observation is implicitly defined here by measurement capable of
selecting alternatives on which we are able to bet (or to gamble ?).
The french word is "parier".

I will not explain the nuance between Bp & Dp and Bp & Dp & p. It just
happen that they are not equivalent in the humble multiverse, that is
they are not equivalent in the discourse of the correct machine, and
this provide nuances. Arithmetical quantum logics appear for all first
person nuances put on the provability predicate (with the comp hyp),
giving three arithmetical interpretation of some quantum logic. Could
explain this latter but I'm afraid it is a bit more technical.

Hoping this could help (to make logician's and physicist's talk closer
perhaps). 'course, Kripke uses himself the term "frame" instead of
multiverse, and "model" when each of the proposition letters (p, q, r,
..) are assigned to true or false (1 or 0, t or f) in each world.

Bruno

http://iridia.ulb.ac.be/~marchal/


Stathis Papaioannou

unread,
Nov 21, 2005, 8:40:44 PM11/21/05
to gl...@quantics.net, everyth...@eskimo.com
George Levy writes:

Can we talk about knowledge or intelligence in a similar way? A rock is
completely stupid and ignorant. A human has some knowledge and some
intelligence (the Goldilocks case). God is said to be omniscient: infinitely
knowlegeable, infinitely intelligent. Doesn't this mean that God is the
equivalent of the blackboard covered in chalk, or the rock?

Stathis Papaioannou

_________________________________________________________________
Start something musical - 15 free ninemsn Music downloads!
http://ninemsn.com.au/share/redir/adTrack.asp?mode=click&clientID=667&referral=HotmailTaglineNov&URL=http://www.ninemsn.com.au/startsomething

Russell Standish

unread,
Nov 21, 2005, 8:52:51 PM11/21/05
to Stathis Papaioannou, gl...@quantics.net, everyth...@eskimo.com
Yes - I believe this is the logical problem with omniscient beings.

--

Kim Jones

unread,
Nov 22, 2005, 1:16:14 AM11/22/05
to Russell Standish, Stathis Papaioannou, gl...@quantics.net, everyth...@eskimo.com
can God in her omniscient, omnipotent wisdom create a rock that is so
heavy even God herself cannot lift it? ;-)

Kim Jones

Russell Standish

unread,
Nov 22, 2005, 1:38:12 AM11/22/05
to Kim Jones, Stathis Papaioannou, gl...@quantics.net, everyth...@eskimo.com
That is the logical argument against omnipotence. IIRC, Aquinas knew
of these arguments, and so I gather omnipotence and omniscience are
not actually part of christian theological creed.

Disclaimer: IANAC (I am not a christian) :)

On Tue, Nov 22, 2005 at 04:57:41PM +1100, Kim Jones wrote:
> can God in her omniscient, omnipotent wisdom create a rock that is so
> heavy even God herself cannot lift it? ;-)
>
> Kim Jones
>

--

Kim Jones

unread,
Nov 22, 2005, 2:15:02 AM11/22/05
to Russell Standish, Stathis Papaioannou, gl...@quantics.net, everyth...@eskimo.com
Wow! So only the Jews and the Muslims can officially rave on about
"G's Omnipotence" etc.

That it?

and IANAC either :)

Sorry - we're gittin off-topic here

he he he

Kim

Quentin Anciaux

unread,
Nov 22, 2005, 2:16:59 AM11/22/05
to everyth...@eskimo.com
Hi, IANAC too, but I think it is part of it.

When I talk about the little paradoxe of the rock to some christians they only
say that the paradoxe is only in the language, that it cannot express what
god is, and that in fact there is no contradiction at all ;)

Jesse Mazer

unread,
Nov 22, 2005, 5:02:27 AM11/22/05
to stathispa...@hotmail.com, everyth...@eskimo.com
Stathis Papaioannou wrote:
>
>George Levy writes:
>
>>Along the line of Jorge Luis Borges a blackboard covered in chalk contains
>>the library of Babel (everything) but no information. Similarly a white
>>board covered with ink also contains no information.
>>Interestingly, information is minimized or actually goes to zero when the
>>world is too large as the plenitude, or too small. Information is
>>maximized when the world is neither too large nor too small. We live in a
>>Goldilock world.
>
>Can we talk about knowledge or intelligence in a similar way? A rock is
>completely stupid and ignorant. A human has some knowledge and some
>intelligence (the Goldilocks case). God is said to be omniscient:
>infinitely knowlegeable, infinitely intelligent. Doesn't this mean that God
>is the equivalent of the blackboard covered in chalk, or the rock?
>
>Stathis Papaioannou

Hmm...but isn't it relevant that an omniscient being is only supposed to
know all *true* information, while the blackboard covered in chalk or
Borges' library would contain all sentences, both true and false? It's like
the difference between the set of all possible grammatical statements about
arithmetic, and the set of all grammatical statements about arithmetic that
are actually true (1+1=2 but not 1+1=3).

Jesse


Jesse Mazer

unread,
Nov 22, 2005, 5:11:08 AM11/22/05
to r.sta...@unsw.edu.au, everyth...@eskimo.com
Russell Standish wrote:

>
>That is the logical argument against omnipotence. IIRC, Aquinas knew
>of these arguments, and so I gather omnipotence and omniscience are
>not actually part of christian theological creed.
>
>Disclaimer: IANAC (I am not a christian) :)

Are you sure about that? I'm pretty sure most christian sects consider God
to be omiscient and omnipotent, although I'm not a christian either. And the
"rock so heavy he can't lift it" isn't too hard to solve if you restrict
omnipotence to that which is logically possible (because it seems the very
definition of omnipotence makes the idea of a rock that can't be lifted by
an omnipotent being self-contradictory).

Jesse


Bruno Marchal

unread,
Nov 22, 2005, 6:17:50 AM11/22/05
to Jesse Mazer, stathispa...@hotmail.com, everyth...@eskimo.com

Le 22-nov.-05, à 10:41, Jesse Mazer a écrit :


I agree. It is a frequent confusion in the list.
Also, people can read the book by Grim "The Incomplete Universe" for a
case that omniscience alone (i.e. without omnipotence) is already
contradictory.

Grim, P. (1991). The Incomplete Universe. The MIT Press, Cambridge, USA

Bruno

http://iridia.ulb.ac.be/~marchal/

George Levy

unread,
Nov 22, 2005, 2:10:05 PM11/22/05
to Everything List
Kim Jones wrote:

> can God in her omniscient, omnipotent wisdom create a rock that is so
> heavy even God herself cannot lift it? ;-)
>
> Kim Jones


Along these lines I found a way to resolve the theist and the atheist
viewpoints: It is simply to assert that god is so big and powerful
(omniscient and omnipotent) that he is the creator of a world capable of
creating itself. Hence his role in creation is nil even though he is the
creator. This approach allows the scientific process to continue as if
there were no god. Evolution is OK. The big bang is OK. It also
satisfies the theists because it raises god to such unimaginable level
of power that he is able create this way: by doing nothing. This god is
surely bigger than any "hands-on-creation" god that classical religions
have come up with. ;-)

George


Russell Standish

unread,
Nov 22, 2005, 5:06:59 PM11/22/05
to George Levy, Everything List
Nice one George!

--

Stephen Paul King

unread,
Nov 22, 2005, 7:32:04 PM11/22/05
to everyth...@eskimo.com
Dear Jesse, Stathis, Bruno et al,

Does this assertion not assume a particular method of coding the "true"
grammatical statements? Could we not show that if we allow for all possible
encodings, symbol systems, etc. that *any* sequence will code a true
statement?

Onward!

Stephen

Jesse Mazer

unread,
Nov 22, 2005, 10:11:28 PM11/22/05
to step...@charter.net, everyth...@eskimo.com


>From: "Stephen Paul King" <step...@charter.net>
>To: <everyth...@eskimo.com>
>Subject: Re: Goldilocks world
>Date: Tue, 22 Nov 2005 19:29:39 -0500

A mathematical platonist would believe that true statements about arithmetic
expressed in a particular language represent platonic truths about
arithmetic that are independent of any particular language you might use to
express them. Anyway, an omniscient being would presumably have a specific
language in mind when judging the truth of any statement made in symbols,
whereas Borges' library or the chalkboard does not specify what language
should be used to interpret a given sequence of symbols.

Jesse


Kim Jones

unread,
Nov 23, 2005, 1:11:51 AM11/23/05
to Russell Standish, George Levy, Everything List
DAMN! Russell replied before me!!

I was going to say - George you're a bloody genius. Stand up and take
a bow. Wait till I tell his Reverend the Archbishop of thingummy this!

He won't believe it I'll bet

"Onward!" as Stephen goes

Kim

Bruno Marchal

unread,
Nov 23, 2005, 6:29:59 AM11/23/05
to jam...@prodigy.net, Everything-List List
Hi John,

Le 22-nov.-05, à 22:45, John M a écrit :

> Bruno:
>
> Why does Jesse - with your aproval - deny from the
> omniscient the knowledge of falls info - maybe WITH
> the notion that it is falls? I am not omniscient -
> believe it or not - but even I know a lot of falls
> info.


By (standard) definition, I would say, no entities (being Gods,
machines, humans, pebbles or extraterrestrials) can know something
false. You can believe something false, but you cannot know something
false. You can know that something is false, but in that case you know
something true. For example you can know that "1+1 = 3" is false, but
in that case you know the *truth* of the proposition " "1+1=3" is
false".
This is reflected in the fact that you will never hear someone saying
"I knew that George Bush was the president of the french republic, but
then I discovered that he was really the president of the USA". Instead
you will hear: "I believed that George Bush was the president of the
french republic, but then I discovered that he was really the president
of the USA". Nobody has ever said "I knew earth is flat but I was
false". The correct sentence is "I believed earth is flat but I was
false".
Indeed this is what has led people from India and China and then Plato
to defined "knowing p" by "believing p and p is true" like the
Theaetetus' first attempt to define knowledge.
Then, the incompleteness phenomena makes those theaetetical nuances
unexpectedly available for the sound machines.


------------------
>
> And 'is' a rock stupid and ignorant indeed?


Who ever said that? Remember my old post (2001):
http://www.mail-archive.com/everyth...@eskimo.com/msg01513.html
You can deduce from it that rocks and pebbles are most probably clever
or "intelligent" according to my oldest theory of intelligence: where
by definition a machine M is intelligent if and only if M is not
stupid, and M is stupid if and only if M believes M is intelligent or M
believes M is stupid. We have good reason to believe that pebbles have
no such beliefs, and this is making them intelligent. You can take this
as a weakness of such a theory, but the cleverness of pebbles here is
just a reflection of the fact that nobody has ever heard a pebbles
communicating some stupidity! I do believe that pebbles are wise and
clever at least in that very general sense. For being stupid, there is
a need of an already non trivial amount of "neural cells".

---------------------------
> Maybe in
> OUR (humanly logical? terms and topics: yes.


You are the one linking "OUR" with humans. I take my "humanity" as a
contingent, accidental, local, and not so interesting fact. More
relevant for the fundamental questions is that I (and we) are most
plausibly descendant of self-duplicating entities.

------------------------
> Do we
> list all unstupidity and knowledgability in the
> totality?

This is already provably impossible for arithmetical truth.

----------------------
> Has anybody ever talked to a rock in rockese?
>
> They wrote big volumes about a (so called) H-atom. Is
> it really perfectly stupid? Holy Anthropocentrism!

Look John, we are perhaps the first having the humility to ask machines
about the fundamental questions and to insist listening to their
answers. Is it possible to be less anthropocentric than that?

Bruno

http://iridia.ulb.ac.be/~marchal/


Bruno Marchal

unread,
Nov 23, 2005, 9:33:42 AM11/23/05
to Stephen Paul King, everyth...@eskimo.com
Hi Stephen,


Le 23-nov.-05, à 01:29, Stephen Paul King a écrit :

> Does this assertion not assume a particular method of coding the
> "true"
> grammatical statements? Could we not show that if we allow for all
> possible
> encodings, symbol systems, etc. that *any* sequence will code a true
> statement?

Sure. It is enough to decide to encode some truth, like "1 = 1" by any
strings. For example the string "6§yhY!!è" will effectively encode "1 =
1".
Now, for any effective coding procedure, you will only get a tiny part
of the true statements of arithmetic, by incompleteness.
And that is why we need to fix the encoding at the start. Then, in any
everything-like theory, we restrict the interpretation by the local
encoding/decoding made by local machines, ...
If not, the only possible TOE will be the inconsistent theory having
all formula as theorem. This does not discriminate anything and could
hardly be considered as providing a theory in the general sense of
scientific theory, given that any facts always confirm it and always
contradict it. It would be like to say that George Bush is the
president of France, adding (after the history teacher makes a
disappointment grin), "oh, but by France I mean that large north
american country". Cool: you will always be right!

Regards,

Bruno


http://iridia.ulb.ac.be/~marchal/


Stathis Papaioannou

unread,
Nov 23, 2005, 8:14:45 PM11/23/05
to laser...@hotmail.com, everyth...@eskimo.com

Jesse Mazer writes:

>>Can we talk about knowledge or intelligence in a similar way? A rock is
>>completely stupid and ignorant. A human has some knowledge and some
>>intelligence (the Goldilocks case). God is said to be omniscient:
>>infinitely knowlegeable, infinitely intelligent. Doesn't this mean that
>>God is the equivalent of the blackboard covered in chalk, or the rock?
>>
>>Stathis Papaioannou
>
>Hmm...but isn't it relevant that an omniscient being is only supposed to
>know all *true* information, while the blackboard covered in chalk or
>Borges' library would contain all sentences, both true and false? It's like
>the difference between the set of all possible grammatical statements about
>arithmetic, and the set of all grammatical statements about arithmetic that
>are actually true (1+1=2 but not 1+1=3).
>
>Jesse

OK, so information = all information, true or false; but knowledge = only
the true information. In that case, we could say that intelligence in an
omniscient being is superfluous, since intelligence could be defined as that
ability which allows one to sort out the true propositions from the false
using certain rules.

On the other hand, this could be too narrow a view of knowledge and
intelligence, restricted to scientific and logical thinking. If we include
artistic creativity, the amount of "knowledge" increases, including abstract
art, abstract literature, every possible musical composition... the
blackboard and the library begin to fill again. It seems that God has to be
a hard-headed scientist who eschews all that artistic nonsense for his
omniscience to be meaningful.

Stathis Papaioannou

_________________________________________________________________
Complimentary Notebook Consultation, courtesy by ASUS
http://a.ninemsn.com.au/b.aspx?URL=http%3A%2F%2Fwww%2Easus%2Ecom%2Eau%2F&_t=752129232&_r=Hotmail_tagline_23Nov05&_m=EXT

Stathis Papaioannou

unread,
Nov 23, 2005, 8:31:18 PM11/23/05
to gl...@quantics.net, everyth...@eskimo.com
George Levy writes:

George's God is perhaps the ultimate expression of deism, the belief that
God made the world but then refrained from any further interference in it.
Most scientists who believe in God should be deists rather than theists if
they are to be consistent, otherwise they would have to include divine
intervention as a possible explanation for every experimental result. This
type of God usually does not satisfy theists, however, because they cling to
the idea that God is personally interested in them, listens to prayers, and
may intervene in the world from time to time if he wishes.

Stathis Papaioannou

unread,
Nov 24, 2005, 2:54:15 AM11/24/05
to mar...@ulb.ac.be, Fabric-o...@yahoogroups.com, everyth...@eskimo.com
Bruno Marchal writes:

>The main idea of Kripke has consisted in saying that the modal formula Bp
>(also written []p) is true at world a, if p is true in all the worlds you
>can access from a. p is relatively necessary at a.
>For example, if the world are countries and if you have to pay taxes in all
>countries that you can access from where you are, then taxes are necessary
>(relatively to a).
>
>That is, p is "necessary" at world a if p is true for all worlds b such
>that aRb. It is intuitively normal: a proposition is necessary for you if
>it is true in all world you can access.

[I have cut this short - Bruno continues at some length from this beginning]

What counts as an accessible world? It seems that in answering this you have
to propose or imply a theory of personal identity. If on the basis of a coin
toss the world splits, and in one branch I am instantaneously killed while
in the other I continue living, there are several possible ways this might
be interpreted from the 1st person viewpoint:

(a) Pr(I live) = Pr(I die) = 0.5

(b) Pr(I live) = 1, Pr(I die) = 0

(c) Pr(I live) = 0, Pr(I die) = 1

Option (c) may look a bit strange but is the one that I favour: all first
person experiences are transient, all branches are dead ends, no world is
accessible from any other world. However, the various independent, transient
observer moments are ordered in such a way in what we experience as ordinary
life that the illusion of (b) occurs. This covers such (theoretical, at
present) cases as the apparent continuity of identity between two observer
moments that just happen to seem to be consecutive "frames" in a person's
life even though there is no physical or informational connection between
them.

Stathis Papaiaonnou

_________________________________________________________________
REALESTATE: biggest buy/rent/share listings
http://ninemsn.realestate.com.au

Bruno Marchal

unread,
Nov 24, 2005, 8:39:29 AM11/24/05
to Stathis Papaioannou, everyth...@eskimo.com, laser...@hotmail.com

Le 24-nov.-05, à 02:06, Stathis Papaioannou a écrit :

>
> Jesse Mazer writes:
>
>>> Can we talk about knowledge or intelligence in a similar way? A rock
>>> is completely stupid and ignorant. A human has some knowledge and
>>> some intelligence (the Goldilocks case). God is said to be
>>> omniscient: infinitely knowlegeable, infinitely intelligent. Doesn't
>>> this mean that God is the equivalent of the blackboard covered in
>>> chalk, or the rock?
>>>
>>> Stathis Papaioannou
>>
>> Hmm...but isn't it relevant that an omniscient being is only supposed
>> to know all *true* information, while the blackboard covered in chalk
>> or Borges' library would contain all sentences, both true and false?
>> It's like the difference between the set of all possible grammatical
>> statements about arithmetic, and the set of all grammatical
>> statements about arithmetic that are actually true (1+1=2 but not
>> 1+1=3).
>>
>> Jesse
>
> OK, so information = all information, true or false;


Mmh... All information is also akin to no information at all (which
explains perhaps why some people makes back and forth between the
notion of everything and nothing).

> but knowledge = only the true information.

Glad to hear that. This illustrates the necessity of agreeing on
definition. I thought nobody would contest that: IF Claude knows p THEN
p is true, by definition of knowledge. If Claude says that she knows
some proposition k, and if it happens later that k appears to be false,
Claude will not say that she knew k, but that she believed k. That is
what the difference between belief and knowledge is all about. In the
modal theories of knowledge we have always the axiom Bp -> p. In the
modal theory of belief we never ask for the axiom Bp -> p. Just because
it makes sense to belief something wrong. But no entities can know
something wrong. If someone believes that we can *know* something
wrong, I would say there is a confusion between the notion of belief
and the notion of knowledge. To insist, if Claude says I know p, and if
you know that p is actually wrong, you will say that Claude believes
something wrong, you will not say that Claude knows something wrong.

I hope everyone agree we take the formula Bp -> p as axioms for a
notion of knowledge.

> In that case, we could say that intelligence in an omniscient being is
> superfluous, since intelligence could be defined as that ability which
> allows one to sort out the true propositions from the false using
> certain rules.


Using rules, or using intuition, memory etc. Personnaly I prefer to use
the weaker term of competence for the ability of making that true/false
discrimination, reserving the word "intelligence" for something deeper
more akin to an open-mindness state or an ability to doubt, etc.
It will appear then that intelligence is necessary for the development
of competence, but that the development of competence has a *negative
feedback" on intelligence.
You can perhaps feel that intuitively: to be very competent can make
you forget that you can be wrong and this could degrade your doubting
ability.


>
> On the other hand, this could be too narrow a view of knowledge and
> intelligence, restricted to scientific and logical thinking.

That is another reason to use the term competence in this setting.
Intelligence is really more like humility or modesty, or wiseness. In
general it is not something which can be evaluated or measured. Only
competence (and even only in circumscribed fields) can be measured.
In school and universities, I think it is a very sad error to confuse
the two. Someone can be very intelligent but completely incompetent.
For example when you have neural problems disallowing your interface
with the world. And the reverse is true too, someone can be very
competent in some field and be completely non-intelligent, incapable of
doubting.

> If we include artistic creativity, the amount of "knowledge"
> increases, including abstract art, abstract literature, every possible
> musical composition... the blackboard and the library begin to fill
> again. It seems that God has to be a hard-headed scientist who eschews
> all that artistic nonsense for his omniscience to be meaningful.


My favorite definition of ...
.. is that thing that once you give it/he/she/e a name or a
description, then you can say "hello" to the catastrophes ....

Bruno


http://iridia.ulb.ac.be/~marchal/


Bruno Marchal

unread,
Nov 24, 2005, 9:38:57 AM11/24/05
to Stathis Papaioannou, Fabric-o...@yahoogroups.com, everyth...@eskimo.com

Le 24-nov.-05, à 08:52, Stathis Papaioannou a écrit :

> Bruno Marchal writes:
>
>> The main idea of Kripke has consisted in saying that the modal
>> formula Bp (also written []p) is true at world a, if p is true in all
>> the worlds you can access from a. p is relatively necessary at a.
>> For example, if the world are countries and if you have to pay taxes
>> in all countries that you can access from where you are, then taxes
>> are necessary (relatively to a).
>>
>> That is, p is "necessary" at world a if p is true for all worlds b
>> such that aRb. It is intuitively normal: a proposition is necessary
>> for you if it is true in all world you can access.
>
> [I have cut this short - Bruno continues at some length from this
> beginning]
>
> What counts as an accessible world? It seems that in answering this
> you have to propose or imply a theory of personal identity.

Actually I would say it is the other way round. Kripke introduces its
abstract "multiverses" in order to be able to make simple the reasoning
for large class of modal logics, which are somehow traditional tools
for handling complex philosophical notions, including notions of
personal identity. That is the way I proceed to. By comp I inherit of
the modal logic G and G* from the most standard theory of
self-reference (the Godel one) and I use them to analyse two (at least)
notions of personal identity (the third person one and the first person
one).


> If on the basis of a coin toss the world splits, and in one branch I
> am instantaneously killed while in the other I continue living, there
> are several possible ways this might be interpreted from the 1st
> person viewpoint:
>
> (a) Pr(I live) = Pr(I die) = 0.5


I hope everyone sees that this (a) is not defensible once we *assume*
comp.

>
> (b) Pr(I live) = 1, Pr(I die) = 0


And this one (b) is a consequence of comp.


>
> (c) Pr(I live) = 0, Pr(I die) = 1
>
> Option (c) may look a bit strange but is the one that I favour: all
> first person experiences are transient, all branches are dead ends, no
> world is accessible from any other world.


I think I figure out why you say that and why you take it probably as a
consequence of comp.
Let us see.

> However, the various independent, transient observer moments are
> ordered in such a way in what we experience as ordinary life that the
> illusion of (b) occurs.

Yes right. But that "illusion" is all what the first person notion is
all about. Your "c" is too strong. What would you say if your comp
doctor proposes you an artificial brain and adds that the Pr(I die),
for you, is 1. I think you would say "no doctor". Then the doctor (not
you!, I know you are doctor!) adds that in all case Pr(I die) = 1. Then
you will tell him that he has not given any clue about the probability
your first person "illusion" (I hate this word) lasts. The real
question we ask to the doctor is what is the probability my "illusion"
will lasts *as* it lasts for any other medical operation when it is
said the operation has been successful.
What I have called "Papaioannou's multiverse" are just your transient
observer moments *together* with the order you are indeed adding on
them for giving sense to ordinary experience. That order *is* an
accessibility relation.

> This covers such (theoretical, at present) cases as the apparent
> continuity of identity between two observer moments that just happen
> to seem to be consecutive "frames" in a person's life even though
> there is no physical or informational connection between them.

But you cannot deny that with comp, there *is* some informational
connection between them. The connection will appear to be exclusively
mathematical and immaterial. And will appear to be the logical root of
another "illusion": a physical world. We know this by UDA (the
Universal Dovetailer Argument), but we need to isolate completely the
structure of the multiverse extractible from comp if we want to derive
the precise physics from comp (and then to compare with the empirical
physics to evaluate empirically the plausibility of comp (or of its
many variants).

Bruno

http://iridia.ulb.ac.be/~marchal/


Stathis Papaioannou

unread,
Nov 24, 2005, 7:12:45 PM11/24/05
to mar...@ulb.ac.be, everyth...@eskimo.com
Bruno Marchal writes:

>>If on the basis of a coin toss the world splits, and in one branch I am
>>instantaneously killed while in the other I continue living, there are
>>several possible ways this might be interpreted from the 1st person
>>viewpoint:
>>
>>(a) Pr(I live) = Pr(I die) = 0.5
>
>
>I hope everyone sees that this (a) is not defensible once we *assume* comp.

Good, we agree here. I don't think everyone on this list would agree.

OK, you've put that quite well. Even if continuity of identity is an
illusion, it is an important illusion. An analogy would be going to the
cinema to see a movie: the "reality" might be that we are watching a series
of still images, but the important thing for the audience is that the
illusion of motion is maintained by having a certain minimum frame rate. So
yes, this does give rise to an accessibility relationship, but it
presupposes a theory of personal identity. Even on this list, there are
people who might say (a) above is the case rather than (b) or (c).

>>This covers such (theoretical, at present) cases as the apparent
>>continuity of identity between two observer moments that just happen to
>>seem to be consecutive "frames" in a person's life even though there is no
>>physical or informational connection between them.
>
>But you cannot deny that with comp, there *is* some informational
>connection between them. The connection will appear to be exclusively
>mathematical and immaterial. And will appear to be the logical root of
>another "illusion": a physical world. We know this by UDA (the Universal
>Dovetailer Argument), but we need to isolate completely the structure of
>the multiverse extractible from comp if we want to derive the precise
>physics from comp (and then to compare with the empirical physics to
>evaluate empirically the plausibility of comp (or of its many variants).

What I meant by "informational connection" was actual information transfer
from one frame to the next, by some physical process. This is what happens
normally by virtue of the fact that consecutive frames are implemented by
the same physical brain. It is also what would happen, in a different way,
with teleportation. This is sufficient for the experience of continuity of
consciousness, but it is not necessary: the appropriate frames or observer
moments might occur completely randomly in different parts of the
multiverse, and the first person experience would be the same. (Such is not
the case for observation of third persons: the frames or observer moments
must be explicitly ordered, or they will be lost in the noise). Is this what
you mean by "the connection will appear to be exclusively mathematical and
immaterial"?

Kim Jones

unread,
Nov 25, 2005, 1:21:13 AM11/25/05
to Bruno Marchal, Stathis Papaioannou, everyth...@eskimo.com, laser...@hotmail.com

On 25/11/2005, at 12:34 AM, Bruno Marchal wrote:

>
>
>
> That is another reason to use the term competence in this setting.
> Intelligence is really more like humility or modesty, or wiseness.

Agreed. People often confuse intelligence with thinking ability (=
operacy, skill at doing)

Edward de Bono defines intelligence as "The horsepower of the car".
It is then, as given a value (probably at birth or not long after)
as, say, the shape of your earlobe or the length of your willy! You
cannot affect your intelligence. You are stuck with it. It is the
measure of the speed at which neurons in your brain fire and receive
impulses. The competency is then the skill with which the car is
driven (skill in thinking, operacy/competency at). The point being:
you can buy a Porsche and drive it badly and kill yourself (woops! I
meant "reduce your measure in the MV") or you can buy a humble
volkswagon and drive it modestly and inexpensively and arrive maybe a
little late.

> In general it is not something which can be evaluated or measured.

Unless you believe in the results of IQ tests. I don't. Why the hell
schools still use them is beyond me

> Only competence (and even only in circumscribed fields) can be
> measured.


Competence is dynamic / intelligence is a frozen quantity of something


> In school and universities, I think it is a very sad error to
> confuse the two. Someone can be very intelligent but completely
> incompetent. For example when you have neural problems disallowing
> your interface with the world. And the reverse is true too, someone
> can be very competent in some field and be completely non-
> intelligent, incapable of doubting.


I can see we share common ground on this


>
>
>
>> If we include artistic creativity, the amount of "knowledge"
>> increases, including abstract art, abstract literature, every
>> possible musical composition... the blackboard and the library
>> begin to fill again. It seems that God has to be a hard-headed
>> scientist who eschews all that artistic nonsense for his
>> omniscience to be meaningful.
>
>
> My favorite definition of ...

> ... is that thing that once you give it/he/she/e a name or a

> description, then you can say "hello" to the catastrophes ....
>

Ain't it "the truth"!

This is also surely because "the truth" is a con job. Truth or
*identity* - which is what you are talking about here - is often the
place at which all movement in thinking ceases. Once you name
something you have slapped a label on it and labels tend to be sticky
things in the warm, spongey human brain. Patterns of recognition act
almost like black holes and suck in all related matter. If something
is said to be *true* then no one thinks much about it anymore and,
more crucially for THIS discussion - all information flow drops to
nil. That's because the Black Hole of Truth has just swallowed up a
whole bunch of creative thinking. Scientists should stay well clear
of truth. Mathematicians own it. This is Bruno's problem in trying to
get maths heads to talk to physics heads.

We have the likes of Socrates, Plato and Aristotle (the Greek Gang of
Three) to thank for a thinking system where simple recognition passes
for thinking. All someone has to do is present something to you as
the truth and 98 per cent will believe it and forget about it. The
reason is quite simply that a pattern of recognition has been placed
in the mind and functions as such (this is the basis of the black
magic of advertising). Edward de Bono has defined a special type of
competency: "Lateral thinking" which is a synonym for creative
thinking. This is where (using formal techniques that can be learnt)
one learns how to cut across the established patterns and make (dare
I say it) a quantum leap to the outcome.

Kim


===========================

A thought once thought cannot be unthought (Edward de Bono)

kimjones@ ozemail.com.au


Bruno Marchal

unread,
Nov 25, 2005, 11:11:31 AM11/25/05
to Stathis Papaioannou, everyth...@eskimo.com

Le 25-nov.-05, à 01:10, Stathis Papaioannou a écrit :

Are you sure?

>
>>> This covers such (theoretical, at present) cases as the apparent
>>> continuity of identity between two observer moments that just happen
>>> to seem to be consecutive "frames" in a person's life even though
>>> there is no physical or informational connection between them.
>>
>> But you cannot deny that with comp, there *is* some informational
>> connection between them. The connection will appear to be exclusively
>> mathematical and immaterial. And will appear to be the logical root
>> of another "illusion": a physical world. We know this by UDA (the
>> Universal Dovetailer Argument), but we need to isolate completely the
>> structure of the multiverse extractible from comp if we want to
>> derive the precise physics from comp (and then to compare with the
>> empirical physics to evaluate empirically the plausibility of comp
>> (or of its many variants).
>
> What I meant by "informational connection" was actual information
> transfer from one frame to the next, by some physical process. This is
> what happens normally by virtue of the fact that consecutive frames
> are implemented by the same physical brain. It is also what would
> happen, in a different way, with teleportation. This is sufficient
> for the experience of continuity of consciousness, but it is not
> necessary: the appropriate frames or observer moments might occur
> completely randomly in different parts of the multiverse, and the
> first person experience would be the same.


Yes. But then I argue that just for that reason it is not necessary
(and actually it is even contradictory) to assume that there is a
physical multiverse. Once you grant the existence of some mathematical
Platonia, then all the appropriate frames or observer moments (relevant
with the comp hyp) occur (partially randomly though) in some
mathematical Platonia. Only from the 1-point of view of the observers
will it be like a movie-life.

> (Such is not the case for observation of third persons: the frames or
> observer moments must be explicitly ordered, or they will be lost in
> the noise).

Indeed and that is exactly what makes comp still possibly false. This
points on the real difficulty of comp: to make sense of the sharable
third person points of view. Quantum mechanics succeeds here thanks to
the non triviality the way the histories sum up (with they destructive
interferences). Now, what I try to convey is that if you ask a
universal machines about the way to sum up their own histories, the
incompleteness phenomena (about which the machine knows a lot) gives a
thorough explanation why the sum is not just the boolean sum of
classical probabilities but a quite quantum-like form of sum.


> Is this what you mean by "the connection will appear to be exclusively
> mathematical and immaterial"?

I am not sure. The universal dovetailer argument (+ movie graph) should
be enough to see that comp makes physics emerging from a mathematical
reality (the set of computations: which is a rare mathematical object
close for the transcendental Cantor-like diagonalization procedures).
The dialog with machines just confirms this, although it does not prove
it of course. I don't think we need the hypothesis of a primitive
physical reality once we can explain its appearance without it. And
any notion of primitive or primary physical reality raises so many
unsolvable questions.


Bruno

http://iridia.ulb.ac.be/~marchal/


Bruno Marchal

unread,
Nov 25, 2005, 12:27:55 PM11/25/05
to Kim Jones, Everything-List List

Le 25-nov.-05, à 07:16, Kim Jones a écrit :

> You cannot affect your intelligence. You are stuck with it. It is the
> measure of the speed at which neurons in your brain fire and receive
> impulses.

From your post I see we agree on many things and I don't need to add
comments, except on this quoted sentence and similar below. I don't
think we are stuck with our intelligence (in the general sense which we
are opposing to competence). Actually I would say that the speed of
processing is more on the side of competence than intelligence
(although I.J. Good makes an interesting analysis of "free-will" in
term of processing speed). Remember I am used to get conversation in
Platonia with Platonic machine which in general are very slow (because
there is no need to optimize them given that in Platonia we have "all
the time"). I think intelligence, again in that large sense of just an
ability of doubting, is very close to courage, and is perhaps just a
matter of attitude. I do think people can get it in one second, but
also to loose it in one second. Generally this happens after some
shock, like when a people you care about dies or when yourself have
some accident or anything which can quickly make fragile some of your
oldest prejudice.


> Competence is dynamic / intelligence is a frozen quantity of something

Same remark. I do think that "intelligence" is the normal state of any
(naive) self-introspective machine. Pain, disease, problematic parents,
problematic social neighborhoods, lack of education etc. all those
rather banal life circumstances can destroy it for a time. And the same
things can also re-awake it (if that is still english).

>>
>> My favorite definition of ...
>> ... is that thing that once you give it/he/she/e a name or a
>> description, then you can say "hello" to the catastrophes ....
>>
>
> Ain't it "the truth"!
>
> This is also surely because "the truth" is a con job. Truth or
> *identity* - which is what you are talking about here - is often the
> place at which all movement in thinking ceases. Once you name
> something you have slapped a label on it and labels tend to be sticky
> things in the warm, spongey human brain.

Yes. And the story of humanity is full of examples. Now it is hard, at
least for me, not to point toward the basic theorems of mathematical
logic in this setting. Tarski theorem: sound löbian machines cannot
name their truth predicate. Gödel's incompleteness theorem: sound
lobian machine cannot prove their own consistency. Now, the lobian
machines, which are just the self-referentially correct machine having
enough introspective power, can prove their Godel's theorem, and so
they can know that if they are consistent they can be inconsistent, and
that is a logical reason for doubting, and that's why I think to be
intelligent is the natural state of a machine, and thus loosing that
intelligence is (alas) also natural. It is like to be alive: to be
alive *is* to be able to die.

Dt -> DBf, to sum up. And that formula characterizes the multiverse
where all transient observer-moment can reach dead-ends. Will come back
to this.

> Scientists should stay well clear of truth. Mathematicians own it.

A scientist has the right to search for the truth, and even to say so.
But he can never be sure it owns it. I'm not sure mathematicians own
it, except perhaps for a tiny part of math, but then everyone owns that
part (except highly disabled person).

Bruno


http://iridia.ulb.ac.be/~marchal/


Stathis Papaioannou

unread,
Nov 26, 2005, 6:53:44 AM11/26/05
to mar...@ulb.ac.be, everyth...@eskimo.com

Stathis Papaioannou writes:
If on the basis of a coin toss the world splits, and in one branch I am
instantaneously killed while in the other I continue living, there are
several possible ways this might be interpreted from the 1st person
viewpoint:

(a) Pr(I live) = Pr(I die) = 0.5

(b) Pr(I live) = 1, Pr(I die) = 0

(c) Pr(I live) = 0, Pr(I die) = 1

Even on this list, there are people who might say (a) above is the case

rather than (b) or (c).

Bruno Marchal replies:
Are you sure?

I was thinking of people who accept some ensemble theory such as MWI, but
don't believe in QTI. I must admit, I find it difficult to understand how
even a dualist might justify (a) as being correct. Would anyone care to
help?

Stathis

Jesse Mazer

unread,
Nov 26, 2005, 1:48:36 PM11/26/05
to stathispa...@hotmail.com, everyth...@eskimo.com

Stathis Papaioannou wrote:

>I was thinking of people who accept some ensemble theory such as MWI, but
>don't believe in QTI. I must admit, I find it difficult to understand how
>even a dualist might justify (a) as being correct. Would anyone care to
>help?

What do you think of my argument here?

http://www.mail-archive.com/everyth...@eskimo.com/msg04692.html


Quentin Anciaux

unread,
Nov 26, 2005, 7:34:18 PM11/26/05
to everyth...@eskimo.com
Le Samedi 26 Novembre 2005 18:47, Jesse Mazer a écrit :
Stathis Papaioannou wrote:
>I was thinking of people who accept some ensemble theory such as MWI, but
>don't believe in QTI. I must admit, I find it difficult to understand how
>even a dualist might justify (a) as being correct. Would anyone care to
>help?

What would be the meaning to accept solution a ?

Are we only sentient entities for a small(art) moment ?

It sounds stupid to only be sentient for a moment... just because a moment has
no meaning for entities like us. Like I like to repeat, what could it means
to not be self aware... ? Could we as first person perspective be "aware" of
not being "aware" ? It sounds non sense.

While I agree it is quite of topic.. this is something that I got lot of
interest into. Why are we looking for a consistent meaning of our own life ?

Quentin

Kim Jones

unread,
Nov 26, 2005, 9:20:53 PM11/26/05
to Quentin Anciaux, everyth...@eskimo.com

On 27/11/2005, at 10:07 AM, Quentin Anciaux wrote:

>
>
> While I agree it is quite of topic.. this is something that I got
> lot of
> interest into. Why are we looking for a consistent meaning of our
> own life ?
>
> Quentin

How can anything be off-topic on a list calling itself
"Everything"????? ;>

Because that's what the human brain does. The nature of the system
called "human brain" involves use of a software (our Greek Gang of
Three judgment-based thinking system) that proceeds on pattern-
recognition for shaping all data. The brain in turn can also supply
patterns of its own. A pattern can be many things of course, but it
in order for our brain to deal with it at all, it has to exhibit some
degree of consistency or regularity, however crazy this may turn out
to be. If you can see meaning in a piece of music then you know what
I mean instantly. If the consistency or regularity don't occur in the
data (or we simply don't percieve it for whatever reason) then our
brains impose a pattern on the data so we can sift it (en faire le
triage).

I always like to say that the absence of real knowledge of something
has never been much of an obstacle to humans tricking up
"explanations" of one kind or another. Without its patterns of
recognition, the brain is a very halting machine.

Also, if we don't actually know something then we can always believe
something, which, it turns out, is almost as good (but not quite as
good).

The search for a "consistent meaning to life" is then the search for
certainty about that pattern one recognises as the 1st person
experience, or the self. I assume that this is not so much for
confirmation of solipsism but for the knowledge that our pattern
counts for something amongst all the others. A kind of emotional
relativity if you will.

The patterns of recognition help us to survive but for what do we
survive? If the white rabbit DOES fly in through the window, then
you've got a problem with the consistency of that meaning. Given half
an hour and a bit of reflection though, you would probably convince
yourself of some explanation. Which is to say you would at least
supply and append (from your own brain) the minimal pattern of
recognition "Gee, if it happened once it could happen again, so I
will suspend my judgment"). Having seen once in my life what I later
came to believe was a UFO, I realise now that for many years I
unconsciously believed some minimal explanation of what I saw
unquestioningly until I really, rationally examined in detail this
"belief". So, whatever we need in the way of consistency of meaning,
we can never be certain we aren't just making it all up as we go
along. I think this brings us back to Bruno and Goedel?

Kim Jones

Saibal Mitra

unread,
Nov 26, 2005, 9:29:27 PM11/26/05
to Stathis Papaioannou, mar...@ulb.ac.be, Jesse Mazer, everyth...@eskimo.com
The answer must be a) because (and here I disagree with Jesse), all that
exists is an ensemble of isolated observer moments. The future, the past,
alternative histories, etc. they all exist in a symmetrical way. It don't
see how some states can be more ''real'' than other states. Of course, the
universe we experience seems to be real to us while alternative universes,
or past or future states of this universe are not being experienced by us.


So, you must think of yourself at any time as being randomly sampled from
the set of all possible observer moments. To get to answer b) you have to
redefine your identity so that experiencing having done the experiment
becomes a necessary part of your identity. But this is cheating because you
wouldn't say that if ''death'' were replaced by a partial memory erasure
such that the experience of having done the experiment were wiped out form
your memory.

Jonathan Colvin

unread,
Nov 27, 2005, 12:56:54 AM11/27/05
to everyth...@eskimo.com
Saibal wrote:
> The answer must be a) because (and here I disagree with
> Jesse), all that exists is an ensemble of isolated observer
> moments. The future, the past, alternative histories, etc.
> they all exist in a symmetrical way. It don't see how some
> states can be more ''real'' than other states. Of course, the
> universe we experience seems to be real to us while
> alternative universes, or past or future states of this
> universe are not being experienced by us.
>
>
> So, you must think of yourself at any time as being randomly
> sampled from the set of all possible observer moments.

<delurk>

I'm not sure how this works. Suppose I consider my state now at time <N> as
a random sample of all observer moments. Now, after having typed this
sentence, I consider my state at time <N + 4 seconds>. Is this also a random
sample on all observer moments? I can do the same at now <N+10>, and so-on.
It seems very unlikely that 3 random samples would coincide so closely. So
in what sense are these states randomly sampled?

Jonathan Colvin

George Levy

unread,
Nov 27, 2005, 1:41:58 AM11/27/05
to Everything List
Stathis Papaioannou wrote:

>
> Stathis Papaioannou writes:
> If on the basis of a coin toss the world splits, and in one branch I
> am instantaneously killed while in the other I continue living, there
> are several possible ways this might be interpreted from the 1st
> person viewpoint:
>
> (a) Pr(I live) = Pr(I die) = 0.5
>
> (b) Pr(I live) = 1, Pr(I die) = 0
>
> (c) Pr(I live) = 0, Pr(I die) = 1


Your example underscores the need for interpreting Pr as a relative
concept ( this is my favorite point of view):
c) is A observing A. It is seen through the first person A who is killed
in one branch and live in another branch. This is called the first
person on this list.
a) is B observing A: It is seen through a first person B who witnesses
the event hapenning to A but lives in both branches. His point of view
is called the third person on this list:
b) is C observing A. It is seen through a first person C who experiences
the complement events of A. He lives when A dies and vice versa. The
probability that he will see A live is 0. We do not have a name for this
point of view on this list but I could suggest "the complement first
person."

Thus all answers are correct depending on your relative point of view.

George

George Levy

unread,
Nov 27, 2005, 2:15:15 AM11/27/05
to Everything List
Please disregard previous post. The b and c cases were inverted.

> Stathis Papaioannou wrote:
>
>>
>> Stathis Papaioannou writes:
>> If on the basis of a coin toss the world splits, and in one branch I
>> am instantaneously killed while in the other I continue living, there
>> are several possible ways this might be interpreted from the 1st
>> person viewpoint:
>>
>> (a) Pr(I live) = Pr(I die) = 0.5
>>
>> (b) Pr(I live) = 1, Pr(I die) = 0
>>
>> (c) Pr(I live) = 0, Pr(I die) = 1
>
>
>
> Your example underscores the need for interpreting Pr as a relative
> concept ( this is my favorite point of view):

> b) is A observing A. It is seen through the first person A who is

> killed in one branch and live in another branch. This is called the
> first person on this list.
> a) is B observing A: It is seen through a first person B who witnesses
> the event hapenning to A but lives in both branches. His point of view
> is called the third person on this list:

> c) is C observing A. It is seen through a first person C who

Stathis Papaioannou

unread,
Nov 27, 2005, 7:08:59 AM11/27/05
to smi...@zeelandnet.nl, mar...@ulb.ac.be, laser...@hotmail.com, everyth...@eskimo.com
Saibal Mitra writes:

>The answer must be a) because (and here I disagree with Jesse), all that
>exists is an ensemble of isolated observer moments. The future, the past,
>alternative histories, etc. they all exist in a symmetrical way. It don't
>see how some states can be more ''real'' than other states. Of course, the
>universe we experience seems to be real to us while alternative universes,
>or past or future states of this universe are not being experienced by us.
>
>So, you must think of yourself at any time as being randomly sampled from
>the set of all possible observer moments. To get to answer b) you have to
>redefine your identity so that experiencing having done the experiment
>becomes a necessary part of your identity. But this is cheating because you
>wouldn't say that if ''death'' were replaced by a partial memory erasure
>such that the experience of having done the experiment were wiped out form
>your memory.
>
>
>

> > Stathis Papaioannou writes:
> > If on the basis of a coin toss the world splits, and in one branch I am
> > instantaneously killed while in the other I continue living, there are
> > several possible ways this might be interpreted from the 1st person
> > viewpoint:
> >
> > (a) Pr(I live) = Pr(I die) = 0.5
> >
> > (b) Pr(I live) = 1, Pr(I die) = 0
> >
> > (c) Pr(I live) = 0, Pr(I die) = 1
> >
> > Even on this list, there are people who might say (a) above is the case
> > rather than (b) or (c).
> >
> > Bruno Marchal replies:
> > Are you sure?
> >
> > I was thinking of people who accept some ensemble theory such as MWI,
>but
> > don't believe in QTI. I must admit, I find it difficult to understand
>how
> > even a dualist might justify (a) as being correct. Would anyone care to
> > help?


I agree that "all that exists is an ensemble of isolated observer moments"
is a good way to look at it. In fact, that is why I think the best
"objective" answer is (c) rather than (b): each OM exists only transiently.
However, as a human, what I am interested in is the experience (one could
say, the illusion) of living my life a step at a time which results from the
existence of certain special OMs in the great and mixed up ensemble of all
possible OMs. Now, where I disagree with you is in the method and meaning of
sampling from this ensemble. It is literally true, in a sense, that my "next
experience" is more likely to be an OM of relatively high measure: a moment
from my life in any month other than November 2005; the experiences of a
Chinese rather than an Australian; death, the content-poor OM of inanimate
matter. If a third person were randomly pulling OMs from the plenitude and
setting them down in order, that is indeed what he would get. Amazingly,
however, when *I* am doing the sampling, my "next experience" always turns
out to be... well, something that we all recognise as a next experience. I
always seem to find that rare OM amongst all the other other ones where I
turn into a turnip, or I'm suddenly 95 years old, or all the other countless
possibilities. I don't even have to go looking for it: if it's out there at
all, I'll find it. If there are several candidate "next moments", including
ones where I have suffered partial memory loss, which one I (that is, one
version of me) experience will seem to be determined probabilistically. And
if there are no candidate next moments at all, then I die.

I have used "I" rather loosely and without defining it because there is no
objective truth of the matter when considering personal identity. I may be
physically completely different (i.e. comprised of different matter) today
than I was a year ago, and my mental state and memories may only be
approximately similar to what they were then, but I am sure I am still the
same person, and that is what counts. If I had undergone a head injury or a
dementing illness in the past year, I would be even less similar now than I
was then, but I would probably still think I was the same person unless I
was really far gone, in which case it probably would be the same as if I had
died. These are matters millions of people deal with every day: you don't
have to bring up multiple copies in other worlds.


Stathis Papaioannou

Stathis Papaioannou

unread,
Nov 27, 2005, 9:41:13 AM11/27/05
to jco...@ican.net, everyth...@eskimo.com
Jonathan Colvin writes:

They would be randomly sampled if, godlike, some third person pulled them
out of the ensemble of all possible observer moments - and in that case, it
certainly would be surprising if these three turned up one after the other.
However, from the first person perspective, they don't need to be "sampled"
at all. It suffices that these three OMs simply exist somewhere in the
plenitude, and - by definition - there will also exist an observer who
experiences (at least) these three states.

The situation is slightly different if there is more than one OM that would
fit a particular state. For example, if there are two distinct OMs that
would fit into the sequence as <N + 4>, then at that point the number of
observers doubles. For the single observer just before the duplication, this
would be seen as a 1/2 probability of experiencing one or the other state.

Stathis Papaioannou

_________________________________________________________________
View 1000s of pictures, profiles and more now at Lavalife
http://lavalife.com.au

Saibal Mitra

unread,
Nov 27, 2005, 10:31:36 AM11/27/05
to Jonathan Colvin, everyth...@eskimo.com

It's a bit like symmetry breaking. You have an ensemble of all possible
observer moment, but each observer moment can only experience its own state.
So, the OM samples itself.

There exists an observer moment representing you at N seconds, at N + 4
seconds and at all possible other states. They all ''just exist'' in the
plenitude, as Stathis wrote. The OM representing you at N + 4 has the
memory of being the OM at N. Subjectively the OMs experience time evolution,
even though the plenitude itself doesn't have a time evolution at the
fundamental level.


Although it is a bit strange to think about time evolution in this way, it
is necessary to resolve paradoxes you get when contemplating doubling and
suicide experiments. It is precisely in these cases that our naive notion of
time evolution breaks down.


Saibal

Brent Meeker

unread,
Nov 27, 2005, 2:51:09 PM11/27/05
to everyth...@eskimo.com, everyth...@eskimo.com

This I find confusing. How is there memory associated with an obserever moment?
Is it equivocation on "memory"? As an experience, remembering something takes
much longer than what I would call "a moment". It may involve a sequence
images, words, and emotions. Of course in a materialist model of the world the
memories are coded in the physical configuration of your brain, even when not
being experienced; but an analysis that takes OM's as fundamental can't refer to
that kind of memories.

Brent Meeker

Jonathan Colvin

unread,
Nov 27, 2005, 5:06:15 PM11/27/05
to everyth...@eskimo.com

I understand all that, but I still don't see in what sense these OM's are
randomly sampled.

Here's a related question. The DDA insists that we must all consider
ourselves random observers on our reference class, whatever it is (class of
all observers is standard). Now, if I am a random observer, and you (Saibal)
are a random observer, what are the odds that two observers selected
randomly from the class of all observers would be discoursing on the same
mailing list? We can only conclude that one of us can not be random, but
must have been selected by the other. But did I select you, or did you
select me? If we select each other, the randomness issue is not resolved.

Another possibility is, I suppose, to simply *define* randomness as observer
self-selection. Perhaps observer self-selection is the only truly random
phenomenon in the universe (everything else appearing random is merely
unpredictable). But it is then a purely a first-person phenomenon, and I can
not consider anything else in the universe (including *your* observer
moments) as random.

Jonathan Colvin

Stathis Papaioannou

unread,
Nov 27, 2005, 6:34:22 PM11/27/05
to laser...@hotmail.com, everyth...@eskimo.com
I agree with everything Jesse says here.

Stathis Papaioannou

_________________________________________________________________

Stathis Papaioannou

unread,
Nov 27, 2005, 9:10:59 PM11/27/05
to meek...@rain.org, everyth...@eskimo.com
Brent Meeker writes:

[quoting Saibal Mitra]


>>There exists an observer moment representing you at N seconds, at N + 4
>>seconds and at all possible other states. They all ''just exist'' in the
>>plenitude, as Stathis wrote. The OM representing you at N + 4 has the
>>memory of being the OM at N.
>
>This I find confusing. How is there memory associated with an obserever
>moment? Is it equivocation on "memory"? As an experience, remembering
>something takes much longer than what I would call "a moment". It may
>involve a sequence images, words, and emotions. Of course in a materialist
>model of the world the memories are coded in the physical configuration of
>your brain, even when not being experienced; but an analysis that takes
>OM's as fundamental can't refer to that kind of memories.

It is true that human cognition, memories etc. are not instantaneous. There
are two ways to keep the OM concept useful despite this. One is to extend
each "moment" so that it encompasses, for example, the minimum period of
awareness (probably a substantial fraction of a second), or any interval of
arbitrary length, such as the waking hours of a day. This still allows one
to think about questions involving continuity of personal identity where
multiple copies or near-copies of a given mind are running simultaneously,
the interval of the OMs under consideration being tailored to the particular
situation. The other way is to bite the bullet and allow instantaneous
part-cognitions. A memory is then only associated with an OM during the act
of remembering, and each instantaneous OM covers only an instant of that
act, in the same way a frame in a film covers only an instant of the action
depicted by the series of frames.

Stathis Papaioannou

_________________________________________________________________
Is your PC infected? Get a FREE online computer virus scan from McAfee®
Security. http://clinic.mcafee.com/clinic/ibuy/campaign.asp?cid=3963

Brent Meeker

unread,
Nov 27, 2005, 9:35:44 PM11/27/05
to everyth...@eskimo.com, everyth...@eskimo.com
Stathis Papaioannou wrote:
> Brent Meeker writes:
>
> [quoting Saibal Mitra]
>
>>> There exists an observer moment representing you at N seconds, at N + 4
>>> seconds and at all possible other states. They all ''just exist'' in the
>>> plenitude, as Stathis wrote. The OM representing you at N + 4 has the
>>> memory of being the OM at N.
>>
>>
>> This I find confusing. How is there memory associated with an
>> obserever moment? Is it equivocation on "memory"? As an experience,
>> remembering something takes much longer than what I would call "a
>> moment". It may involve a sequence images, words, and emotions. Of
>> course in a materialist model of the world the memories are coded in
>> the physical configuration of your brain, even when not being
>> experienced; but an analysis that takes OM's as fundamental can't
>> refer to that kind of memories.
>
>
> It is true that human cognition, memories etc. are not instantaneous.
> There are two ways to keep the OM concept useful despite this. One is to
> extend each "moment" so that it encompasses, for example, the minimum
> period of awareness (probably a substantial fraction of a second), or
> any interval of arbitrary length, such as the waking hours of a day.
> This still allows one to think about questions involving continuity of
> personal identity where multiple copies or near-copies of a given mind
> are running simultaneously, the interval of the OMs under consideration
> being tailored to the particular situation.

But giving OMs duration seems to invite other incoherence. It means that time
cannot be understood as a sequence of timeless OMs. On the other hand it solves
more than just the memory problem; if OMs have duration, then the durations
could overlap and thus define "worlds" and "personal identity" - i.e. provide
the accessiblity relation.

>The other way is to bite the
> bullet and allow instantaneous part-cognitions. A memory is then only
> associated with an OM during the act of remembering, and each
> instantaneous OM covers only an instant of that act, in the same way a
> frame in a film covers only an instant of the action depicted by the
> series of frames.

I have difficultly with an "instant of cognition". A film records an instant of
spatial relations, but how is one to understand a non-extensive, instant of
cognition - certainly not by simple introspection. But it seems that getting
an explanation of the world via introspection is why OMs were appealing in the
first place.

Brent Meeker

Bruno Marchal

unread,
Nov 28, 2005, 11:08:13 AM11/28/05
to jam...@prodigy.net, Everything-List List

Hi John,

> Bruno,Hi,
> thanks for your speedy and considerate answer. Your
> examples are so simplistic, as only the science of
> logic can provide. Let me try better examples:
>
> --- Bruno Marchal <mar...@ulb.ac.be> wrote:
>
> " For example you can know that "1+1 = 3" is false,
> but
>> in that case you know the *truth* of the proposition
>> " "1+1=3" is false".
>
> Could we figure anything beyond 101 elementary school
> arithmetics?


Why? The point is just to agree (or agree that we disagree) on some
definition. The point is that nobody can know "something false", as
opposed to knowing that some proposition is false. It is better to
illustrate such nuance with elementary propositions nobody (really)
doubt about (of the kind 1+1 = 2, or "1+1 = 3" is false).


> E.g.: People and scientists, priests, etc. KNEW for
> many centuries, including the early medieval ages that
> the Earth is the center of the world and everything
> rotates around it.


They knew that? Or they believed that ? I don't think it makes sense
to say they knew that, unless you have another definition of knowledge.
It is up to you to explain the difference between belief and knowledge.
Epistemologist generally agree that knowledge verify the

Bp -> p

formula (If I know p then p is true). This is just because the truth is
put in the knowledge by definition. If not, it means we talk on
beliefs. Of course many people pretend that they know some proposition,
and occasionally they are wrong: but then they say (if honest): "ok I
was wrong but I believed it", they does not say "ok I was wrong but I
knew it".
And I do think our definition should be coherent with the way we talk,
unless there is a big reason to depart from the traditional use.


> Then came Copernicus and said: this
> is wrong, the sun is the middlepoint. And people
> though reluctantly, believed it. Then came
> cosmologists and said that is wrong, there is NO
> center, everything - including the Sun is moving
> around. And people believed it finally, in droves.
> Until Eistein came around and procalimed: nothing
> moves around anything, because all movement is
> relative to the others, ---
> ACCORDINGLY:
> the notion that everything rotates around the Earth is
> just as true as any other belief put forward ever
> since. (Warum habe wir die Kröten gefressen?)


But this oscillation is a quite complex things, and the usage of the
word "knowing" here is quite sophisticated (that is why I did limit
myself to very elementary arithmetic exemple).
All what I can say is that if the earh is at the center of the world
then the priest knew it, and poor of us, we believe (wrongly) the
contrary. If the earth is not at the center of the world then the
priests did believed (wrongly) that, and we know better.
But your example includes a notion of relativity which limits its use
for making clear the difference between the notion of belief and
knowledge.


> Popper had the idea that
> nothing in science can be proven as true, only
> falsification is possible, does that mean that all
> science is false?


It just means that all (empirical) science is uncertain.
Actually a large part of analytical science is also uncertain but for
different reasons (and then with comp it can be shown that there are
relations between those different sort of uncertainties), but it would
be senseless to mention them before we agree on the basic vocabulary.


Bruno


http://iridia.ulb.ac.be/~marchal/


Bruno Marchal

unread,
Nov 28, 2005, 11:16:30 AM11/28/05
to Quentin Anciaux, everyth...@eskimo.com

Le 27-nov.-05, à 00:07, Quentin Anciaux a écrit :

> Why are we looking for a consistent meaning of our own life ?

What would be an inconsistent meaning? (i'm just trying to figure out
what you ask)


Bruno

http://iridia.ulb.ac.be/~marchal/


Bruno Marchal

unread,
Nov 28, 2005, 11:23:10 AM11/28/05
to Quentin Anciaux, Everything-List List

Le 27-nov.-05, à 02:18, Kim Jones a écrit :

> The search for a "consistent meaning to life" is then the search for
> certainty about that pattern one recognises as the 1st person
> experience, or the self. I assume that this is not so much for
> confirmation of solipsism but for the knowledge that our pattern
> counts for something amongst all the others. A kind of emotional
> relativity if you will.

And that can be explained in some Darwinian way (once we postulated
some amount of consistency around us).

OK I should have read the two posts. I think I understand better what
you are aiming to.

Bruno

http://iridia.ulb.ac.be/~marchal/


Bruno Marchal

unread,
Nov 28, 2005, 11:52:47 AM11/28/05
to Saibal Mitra, Jesse Mazer, Stathis Papaioannou, everyth...@eskimo.com

Le 27-nov.-05, à 02:25, Saibal Mitra a écrit :

> The answer must be a) because (and here I disagree with Jesse), all
> that
> exists is an ensemble of isolated observer moments. The future, the
> past,
> alternative histories, etc. they all exist in a symmetrical way. It
> don't
> see how some states can be more ''real'' than other states.

But then how could we ever explain why some states seem to be more
*near*, or more probable than others from our point of view?

Is the choice between Papaioannou's "a", "b" reflecting(*) the ASSA
and RSSA difference?

Recall: ASSA = absolute self-sampling assumption. RSSA = relative
self-sampling assumption.

(*) Stathis Papaioannou writes:
> If on the basis of a coin toss the world splits, and in one branch I am
> instantaneously killed while in the other I continue living, there are
> several possible ways this might be interpreted from the 1st person
> viewpoint:
>
> (a) Pr(I live) = Pr(I die) = 0.5
>
> (b) Pr(I live) = 1, Pr(I die) = 0
>
> (c) Pr(I live) = 0, Pr(I die) = 1
>
> Even on this list, there are people who might say (a) above is the case
> rather than (b) or (c).


Saibal:

> So, you must think of yourself at any time as being randomly sampled
> from
> the set of all possible observer moments.


This could make sense in a pure third person perspective, but then it
is no more a perspective. And, indeed, to predict the result of
anything I decide to test, I need to take into account relations
between observer-moments. Let me throw a dice. Are you saying to us
that to predict the result I need to take into account all
observer-moments and sample on them in some "uniform" way. Why should
people buy lotto-tickets? They could make the big win by their OM being
sampled on all OMs.
I'm not saying you are false, but your absolute sample does not
correspond tour first person experience (including physics) which we
want to explain. It seems to me.

> To get to answer b) you have to
> redefine your identity so that experiencing having done the experiment
> becomes a necessary part of your identity.

Not some absolute identity, but memories are part of our relative,
mundane, identity.

> But this is cheating because you
> wouldn't say that if ''death'' were replaced by a partial memory
> erasure
> such that the experience of having done the experiment were wiped out
> form
> your memory.

OK, but that is why the experiment is proposed with (absolute) death
(if that exists) and not with memory erasure. This could change the
probabilities a lot, and this can admit many different protocol for
verifying the probability distributions. It is another experiment.
Perhaps I miss your point.

Bruno

http://iridia.ulb.ac.be/~marchal/


Stathis Papaioannou

unread,
Nov 28, 2005, 7:31:17 PM11/28/05
to smi...@zeelandnet.nl, mar...@ulb.ac.be, laser...@hotmail.com, everyth...@eskimo.com
Saibal Mitra writes:
>The answer must be a) because (and here I disagree with Jesse), all that
>exists is an ensemble of isolated observer moments. The future, the past,
>alternative histories, etc. they all exist in a symmetrical way. It don't
>see how some states can be more ''real'' than other states. Of course, the
>universe we experience seems to be real to us while alternative universes,
>or past or future states of this universe are not being experienced by us.

> > Stathis Papaioannou writes:


> > If on the basis of a coin toss the world splits, and in one branch I am
> > instantaneously killed while in the other I continue living, there are
> > several possible ways this might be interpreted from the 1st person
> > viewpoint:
> >
> > (a) Pr(I live) = Pr(I die) = 0.5
> >
> > (b) Pr(I live) = 1, Pr(I die) = 0
> >
> > (c) Pr(I live) = 0, Pr(I die) = 1

Another thought: if I die instantaneously in one of the two branches - that
is, I don't have time to experience that branch at all - is this not
functionally equivalent to being copied and instantaneously killed in
multiple branches? In the next moment, I expect to find myself alive and
continuing to type this in Melbourne, but dead in Sydney, Paris, Mars etc. -
dead almost everywhere else in the multiverse, in fact. Given the reasoning
in support of answer (a), doesn't this mean I should have almost zero
expectation of finding myself alive in Melbourne in the next moment?

Stathis Papaioannou

unread,
Nov 29, 2005, 2:32:09 AM11/29/05
to meek...@rain.org, everyth...@eskimo.com

An observer a1 at time t1 undergoes destructive scanning, and two exact
copies, observers a2 and a3, are created. If we ask a2 and a3, they will
each claim to remember "being" a1. We could say that as a result of the
duplication we have two people, a1a2 and a1a3, each with equal claim to have
been the original person a1.

Now, consider the use of the terms "observer" and "person" in the above
paragraph. In an old-fashioned linear world where a person is born, lives
his life, and dies, there is little point in making distinctions between the
two terms. When branchings and duplications start happening, however, the
concept of "person" becomes complicated and ambiguous. In the given example,
the observers a1, a2, a3 are unambiguous, separated from each other in time
and space; and it is this basic entity which I call an observer moment. A
person is then a complex entity extending along a certain path in the
multiverse, comprised of a concatenation of observer moments.

The observer moments a1, a2, a3 as defined above have duration and are
delimited by the duplication event at t1. There is no reason why the
duplication could not happen while a1 is in mid-thought. In that case, a2
and a3 may or may not continue and complete the thought, or they may
complete different thoughts (that start the same way) given that a2 and a3
may now be in different environments. It would then be a matter of
definition whether we say the thought cut short at t1 was half in a1 and
half in a2/a3, or whether we say it was completely in a2/a3 since, as I
think you are arguing, half a thought does not make a conscious experience.
It would be technically simpler if we allowed for part-thoughts, even though
this detracts somewhat from the intuitive appeal of the observer moment
concept. This would also allow for a continuum of instantaneous observer
moments, consistent with the continuous branching presumed to occur in the
multiverse.

Stathis Papaioannou


Brent Meeker writes (quoting Stathis):

_________________________________________________________________
On the road to retirement? Check out MSN Life Events for advice on how to
get there! http://lifeevents.msn.com/category.aspx?cid=Retirement

uv

unread,
Nov 30, 2005, 7:04:22 PM11/30/05
to Everything List
George Levy originally said

> Interestingly, information is minimized or actually goes to zero
> when the world is too large as the plenitude, or too small.
> Information is maximized when the world is neither too large
> nor too small. We live in a Goldilock world.

This is a fine concept and one of the great powers of many-worlds is
that they can allow us to introduce such new concepts into our
thinking. I did not mention Goldilocks worlds in my current blog at
http://ttjohn.blogspot.com/
or egroup at t...@yahoogroups.com (moderated posting possible
on some many worlds topics) yet but I covered Harry Potter
worlds and the Deutsch interpretation up to a point this week
with more to follow soon on category theory and possibly
Goldilocks worlds will be dealt with.

uv


Stathis Papaioannou

unread,
Dec 1, 2005, 1:20:00 AM12/1/05
to meek...@rain.org, everyth...@eskimo.com

Why does an OM need to contain so much information to link it to other OMs
making up a person? I certainly don't spend every waking moment reminding
myself of who I am, let alone going over my entire past history, and I still
think all my thoughts are my thoughts. I don't think that the fact these
thoughts are contained in my head makes the difference, because as you
seemed to agree, continuity of consciousness can in theory extend over
discontinuities in time and/or space, as in teleportation. On the other
hand, I could suddenly become psychotic and as a result believe I am a
completely different person, with a different past; or perhaps my mind could
be taken over by an alien intelligence with a similar effect. As for one OM
potentially representing a thought from thousands of different people, that
is exactly what happens in the multiverse and is one of the key advantages
of the concept. Suppose you and I happened to have *exactly* the same
subjective experience at a particular time, say seeing a red shape on a
white wall at the age of two. This would mean that, for that moment, your
mind and my mind could have been interchanged, or one of our two minds could
have been temporarily suspended, without making any subjective difference to
either of us. An external observer monitoring my body might have noticed a
momentary blankness if my mental processes were suspended at the moment of
coincidence, but as far as I was concerned, it would have been exactly the
same as if I were teleported away to have the red shape experience (which I
would have had anyway) and teleported back.

Stathis Papaioannou

>Stathis Papaioannou wrote:
>>Brent Meeker writes:
>>

>>>I agree with all you have written below as an explication of what we mean
>>>by a
>>>person in the multiverse. But it assumes an objective spacetime in order
>>>to
>>>define persons by causal continuity. I thought the point of OMs was to
>>>provide
>>>a fundamental ontology from which spacetime would be constructed.
>>
>>
>>While it always seems in real life, as in my example, that there is a
>>causal connection between related OMs, this need not necessarily be the
>>case. For a2 to think, "I stepped into the teleporter a moment ago" and to
>>consider himself the person a1a2, it is sufficient simply that a2 exist.
>>That is, given that a2 exists, it makes no difference whether there is
>>information transfer from a1 to a2, whether a1 precedes a2, or whether a1
>>exists at all. In general, if the only thing that exists is the set of all
>>possible OMs, not ordered in any particular way and each OM completely
>>independent and isolated, then the apparent multiverse with its complex
>>physical laws results as an emergent phenomenon, or if you prefer, an
>>illusion.
>
>That's taking an OM to be like Barbour's time capsule. They are ordered
>according to their contents. a1a2's OM with the thought that he was in a
>teleporter and was a1, connects to a1's OM with the thought that now I am
>stepping into a teleporter. But that brings me back to my objection to
>OMs.
>Barbour's time capsules contain whole states of the world. OMs don't have
>enough information to provide the specificity required for connections.
>a1a2's
>OM thought could be the thought of thousands of other people. To be sure,
>if we
>take a sequence of a1a2's thoughts and lump them into one OM then that OM
>will
>have enough information to place in a unique sequence indentifying a
>person.
>But then we've really assumed the thing to be explained.
>
>Brent Meeker

Bruno Marchal

unread,
Dec 2, 2005, 9:01:35 AM12/2/05
to Stathis Papaioannou, meek...@rain.org, everyth...@eskimo.com

Le 01-déc.-05, à 07:17, Stathis Papaioannou a écrit :

> Why does an OM need to contain so much information to link it to other

> OMs making up a person? [the complete message is below].


I am not sure I understand. Are you saying, like Saibal Mitra, that OMs
(Observer-Moments) are not related? How, in this case, would you
interpret your own talk about "next observer moment" (those which could
be dead end)?
Is there not a confusion between the idea of physicalist (causal) view
of the relation between OMs (which, as Brent meeker said should be
explained from a more primitive (mathematical, immaterial, not causal,
..) notion of OM, with those very (more primitive) OMs.
Are you assuming some notion of multiverse richer than (or just
different from) a notion of multi-OMs?

At least, when you "interview" a sound lobian machine on such questions
(through the modal logic G), or better when you interview its "guardian
angel" (through its modal logic G*), you can understand that the
"ultimate" multiverse can reasonably be said not having structure, and
that multiverse-structures *appear* for each notion of self-referential
points of view (not necessarily first person pov). The first person pov
makes the "multiverse" a temporal structure, the first person plural
pov makes the multiverse a quantum probability structure.


***

Mmmmmhhhhhh.... I know this could look like jargon. Let me give "easy"
exercises for anybody following this list.

Let me define a Multiverse (called also "frame" by Kripke) as any
non-empty set W together with an accessibility relation R defined on
the set. Elements of that set are called "world", by definition, and I
follow the convention to denote worlds by greek letters (or their
english transcription: alpha, beta, gamma, delta, eta, epsilon, iota,
kappa, omega, nu, theta, etc.). R is called the accessibility relation.
So the simplest example of multiverse is given by the set {alpha} +
the empty relation (so just one dead end!). Another example is the set
of natural numbers with the divisibility relation ( n R m iff n divide
m iff there is a k such that n * k = m).


Let me define a notion of illuminated multiverse (called "model" by the
modal logicians). It is just a Kripke multiverse where we associate to
each world a value 1 or 0 to each sentence letter. The Kripke
multiverse is "illuminated" when a truth value (1 or 0) is assigned to
each proposition, in each world.
Remember that in (propositional) logic we have sentence letter p, q, r,
etc.
We also say that p is true in alpha for p has value 1 in alpha (in some
illuminated multiverse).


Now Kripke semantics can be given in a very simple way, by just asking
that,

1) each world obeys to classical logic (that is: if 1 is assigned to p
in the world alpha, and if 1 is assigned to q in alpha, then 1 is
assigned to (p & q) in alpha, etc. The "etc" is just a pointer to the
usual truth table of classical propositional logic. I have already
explain this on this list but I can do it again if asked). In
particular each classical tautologies are true in all worlds, whatver
the illumination chosen (whatever the truth value of the sentence
letter are in each world: like (p -> p) or (p v ~p), etc.

2) Kripke says that Bp (also written box p, []p, etc.) is true in the
world alpha if p is true in all worlds beta accessible from alpha. From
this it follows that Dp (defined as an abbreviation of ~B~p) will be
true in some world alpha if there is some world beta, accessible from
alpha, and with p true in the world beta.

Now I will say that a formula A of modal logic is valid in a
illuminated multiverse (W, R, V) if A is true in all the worlds of
that illuminated multiverse.

And I will say that a formula A of modal logic is respected by a
multiverse (W,R) if A is valid
in all illuminated multiverse (W, R, V). Or equivalently: A is
respected in (W,R) if A is true in all worlds in W and this for all
"illuminations" V, i.e. for all assignment of truth value of the
sentence letters in all worlds.

Last definition: a multiverse (W,R) is said to be reflexive if the
relation R is reflexive (that is: if for all world in W we have xRx,
i.e. if each world is accessible to itself by the relation R.

The easy exercise is the following: show that if the multiverse is
reflexive then the multiverse respects the formula Bp -> p.

Slightly less easy: show that the reverse is true: show that if a
multiverse respects Bp -> p, then the multiverse is reflexive.

I would like to know if that exercise *seems* difficult. For those who
cannot do it, it just means there is a need to refresh some "naive set
theory" knowledge, and I will think about a book who can help.

Don't hesitate to answer out of line if you prefer.

Sorry to annoy you with that modal stuff, but we are at a point I could
no more comment the posts without making nuances which will resemble
jargon if you don't invest a little bit in modal logic. This, by the
way, could provide us with a language capable of make clearer many
other "everything-like philosophy clearer.


Bruno


The complete original message by Stathis Papaioannou:

http://iridia.ulb.ac.be/~marchal/


Saibal Mitra

unread,
Dec 2, 2005, 7:29:34 PM12/2/05
to Brent Meeker, everyth...@eskimo.com


Well, what really matters is that the laws of physics define a probability
distribution over OMs. So, there is no problem thinking of yourself as being
sampled randomly from that probability distribution. The length of an OM
can be taken to be zero. Even if recalling something takes time, at any time
you are at a certain point in that process. There exists an OM that recalls
going through that sequence, but that is also at a specific moment in time.

Saibal Mitra

unread,
Dec 2, 2005, 7:47:26 PM12/2/05
to Jonathan Colvin, everyth...@eskimo.com
----- Original Message -----
From: "Jonathan Colvin" <jco...@ican.net>
To: <everyth...@eskimo.com>
Sent: Sunday, November 27, 2005 10:02 PM
Subject: RE: Quantum Immortality and Information Flow


>


Yes, I meant ''random'' in the sense of observer self selection. But note
that the laws of physics define, in principle, a probability distribution
over the set over all possible states you can be in. One element of that set
corresponds to you reading this sentence. The probability of this is given
by an integral of the probability of states of the universe that are
consistent with you experiencing this OM. So, you ''integrate out''
everything that is not part of the OM and you are left with the probability
of the OM.

Saibal Mitra

unread,
Dec 2, 2005, 7:56:24 PM12/2/05
to Bruno Marchal, Jesse Mazer, Stathis Papaioannou, everyth...@eskimo.com

----- Original Message -----
From: "Bruno Marchal" <mar...@ulb.ac.be>
To: "Saibal Mitra" <smi...@zeelandnet.nl>
Cc: "Jesse Mazer" <laser...@hotmail.com>; "Stathis Papaioannou"
<stathispa...@hotmail.com>; <everyth...@eskimo.com>
Sent: Monday, November 28, 2005 04:47 PM
Subject: Re: Quantum Immortality and Information Flow


>


> Le 27-nov.-05, à 02:25, Saibal Mitra a écrit :
>
> > The answer must be a) because (and here I disagree with Jesse), all
> > that
> > exists is an ensemble of isolated observer moments. The future, the
> > past,
> > alternative histories, etc. they all exist in a symmetrical way. It
> > don't
> > see how some states can be more ''real'' than other states.
>
> But then how could we ever explain why some states seem to be more
> *near*, or more probable than others from our point of view?

Well, even if you assume ''ordinary'' laws of physics, you can have this
view. Einstein tried to console a friend whose son had died, by saying that
although he isn't alive now, he is ''still'' alive in the past. Relativity
theory threats space and time in more or less symmetrical ways.It doesn't
make any difference if you assume that you are sampled from a probability
distribution (to be calculated from physics) over your experiences.

Well, the probability distribution has to be consistent with physics. In
case of throwing a dice, one should consider the set of OMs that are
experiencing the outcome of the throw.


>
>
>
> > To get to answer b) you have to
> > redefine your identity so that experiencing having done the experiment
> > becomes a necessary part of your identity.
>
> Not some absolute identity, but memories are part of our relative,
> mundane, identity.
>
>
>
> > But this is cheating because you
> > wouldn't say that if ''death'' were replaced by a partial memory
> > erasure
> > such that the experience of having done the experiment were wiped out
> > form
> > your memory.
>
> OK, but that is why the experiment is proposed with (absolute) death
> (if that exists) and not with memory erasure. This could change the
> probabilities a lot, and this can admit many different protocol for
> verifying the probability distributions. It is another experiment.
> Perhaps I miss your point.


Yes, that was my point. The probabilities become sensitive to the details of
the set up in a way that I find unphysical. If we just do conventional
quantum measurement of z-component of a spin polarized in the x-direction.
Then, in the MWI, you would say that there exists a world in which an
observer sees spin up and a world in which spin down is experienced.
Strictly speaking the two observers are not identical. Let's now modify the
experiment so that in case of spin down the observer is annihilated and
replaced by some arbitrary person. Then if we choose this person to be
''close'' to the original person then the probabilities are 1/2, but if I
move sufficiently ''far away'' from the person then it should somehow jump
to 1 for the original person.

Saibal Mitra

unread,
Dec 2, 2005, 8:20:25 PM12/2/05
to everything
Correction, I seem to have misunderstood Statis' set up. If you really
create a new world and then create and kill the person there then the
probability of survival is 1. This is different from quantum mechanical
branch splitting.

To see this, consider first what would have happened had the person not been
killed. Then his measure would have doubled. But because he is killed in one
of the two copies of Earth, his measure stays the same. In a quantum suicide
experiment his measure would be reduced by a factor two.

George Levy

unread,
Dec 2, 2005, 11:11:59 PM12/2/05
to Everything List
Saibal Mitra wrote:
Correction, I seem to have misunderstood Statis'  set up. If you really
create a new world and then create and kill the person there then the
probability of survival is 1. This is different from quantum mechanical
branch splitting.

To see this, consider first what would have happened had the person not been
killed. Then his measure would have doubled. But because he is killed in one
of the two copies of Earth, his measure stays the same. In a quantum suicide
experiment his measure would be reduced by a factor two.
  
To say that measure is doubled or halved it is not sufficient to take the measure at the final point. You really must compare measure at two points, in effect take a ratio. So depending where the initial point is you could come to different conclusions. If your initial point is before the new world is created (and the clone in that world is killed), then, you are right. There is no change in the measure of the original person. However, if the initial point is taken after the world is created but before the clone is killed, then the measure of the clone goes to zero "in that world." One could always argue that the world branches and the clone continues living in other worlds.

George

Quentin Anciaux

unread,
Dec 3, 2005, 4:02:09 AM12/3/05
to everyth...@eskimo.com
Hi Saibal,

Le Samedi 3 Décembre 2005 02:15, Saibal Mitra a écrit :
> Correction, I seem to have misunderstood Statis' set up. If you really
> create a new world and then create and kill the person there then the
> probability of survival is 1. This is different from quantum mechanical
> branch splitting.
>
> To see this, consider first what would have happened had the person not
> been killed. Then his measure would have doubled. But because he is killed
> in one of the two copies of Earth, his measure stays the same. In a quantum
> suicide experiment his measure would be reduced by a factor two.

His measure would be reduced by a factor two relative to what ? Do you mean
there exists an absolute measure ?

Quentin


Russell Standish

unread,
Dec 3, 2005, 5:08:52 AM12/3/05
to Bruno Marchal, Fabric-o...@yahoogroups.com, everyth...@eskimo.com
On Mon, Nov 21, 2005 at 03:39:58PM +0100, Bruno Marchal wrote:
> Observation is implicitly defined here by measurement capable of
> selecting alternatives on which we are able to bet (or to gamble ?).
> The french word is "parier".
>

Well at least this isn't a problem of translation. But I still have
difficulty in understanding why Pp=Bp & -B-p should be translated into
English as "to bet on p" (or for that matter pourquoi on devrait
le traduire par "a parier a p")

For me Bp & -B-p is simply a statement of consistency - perhaps what
we mean by mathematical truth.

Cheers

--
*PS: A number of people ask me about the attachment to my email, which
is of type "application/pgp-signature". Don't worry, it is not a
virus. It is an electronic signature, that may be used to verify this
email came from me if you have PGP or GPG installed. Otherwise, you
may safely ignore this attachment.

----------------------------------------------------------------------------
A/Prof Russell Standish Phone 8308 3119 (mobile)
Mathematics 0425 253119 (")
UNSW SYDNEY 2052 R.Sta...@unsw.edu.au
Australia http://parallel.hpc.unsw.edu.au/rks
International prefix +612, Interstate prefix 02
----------------------------------------------------------------------------

Stathis Papaioannou

unread,
Dec 3, 2005, 5:41:16 AM12/3/05
to mar...@ulb.ac.be, meek...@rain.org, everyth...@eskimo.com
Bruno Marchal writes:

>Le 01-déc.-05, à 07:17, Stathis Papaioannou a écrit :
>
>>Why does an OM need to contain so much information to link it to other OMs
>>making up a person? [the complete message is below].
>
>
>I am not sure I understand. Are you saying, like Saibal Mitra, that OMs
>(Observer-Moments) are not related? How, in this case, would you interpret
>your own talk about "next observer moment" (those which could be dead end)?
>Is there not a confusion between the idea of physicalist (causal) view of
>the relation between OMs (which, as Brent meeker said should be explained

>from a more primitive (mathematical, immaterial, not causal, ...) notion

>of OM, with those very (more primitive) OMs.
>Are you assuming some notion of multiverse richer than (or just different
>from) a notion of multi-OMs?

In our ordinary experience, the OMs making up an individual's stream of
consciousness are causally related by virtue of the fact that that they
occur inside the same brain. If we consider thought experiments involving
teleportation or mind uploading, again the sequential OMs are causally
related due to transfer of the relevant brain pattern (or whatever)
information. However, this information tranfer is not actually *necessary*
for the OMs to be experienced as moments in the same stream of
consciousness. Say an observer experiencing OM a1 enters a teleporter which
then causes another observer experiencing OM a2 to be created at the
receiving station. Then a1 and a2 are sequential OMs, constituting a stream
of consciousness a1a2 sampled from the life of an individual. If this is so,
then if a1 occurs anywhere in the multiverse, and a2 occurs anywhere else,
the same stream of consciousness a1a2 should be experienced - even if a1 and
a2 occur completely at random, with no "causal link" between them.

I am agnostic regarding the question of whether OMs are primitive or
derivative. The world could be as it appears: the physical universe
(whatever that means) gives rise to certain special physical processes which
result in moments of conscious experience, and those moments which are
related through being the product of circumscribed subsets of physical
processes constitute a stream of consciousness in an individual life. On the
other hand, in a world where exactly the same OMs as postulated in the
previous sentence exist, but all mixed up and not connected to any (or any
particular) physical process, exactly the same individual streams of
consciousness would result.

Stathis Papaioannou

Stathis Papaioannou

unread,
Dec 3, 2005, 6:08:36 AM12/3/05
to smi...@zeelandnet.nl, everyth...@eskimo.com
Well, I did actually intend my example to be analogous to the Tegmark QS
experiment. Are you saying that if there is only one world and magically an
identical, separate world comes into being this is fundamentally different
to what happens in quantum branch splitting? It seems to me that in both
cases the relative measure of everything in the world stays the same, even
though in absolute terms there is double of everything.

Stathis Papaioannou


Saibal Mitra writes:

>Correction, I seem to have misunderstood Statis' set up. If you really
>create a new world and then create and kill the person there then the
>probability of survival is 1. This is different from quantum mechanical
>branch splitting.
>
>To see this, consider first what would have happened had the person not
>been
>killed. Then his measure would have doubled. But because he is killed in
>one
>of the two copies of Earth, his measure stays the same. In a quantum
>suicide
>experiment his measure would be reduced by a factor two.

> > If on the basis of a coin toss the world splits, and in one branch I am


> > instantaneously killed while in the other I continue living, there are
> > several possible ways this might be interpreted from the 1st person
> > viewpoint:
> >
> > (a) Pr(I live) = Pr(I die) = 0.5
> >
> > (b) Pr(I live) = 1, Pr(I die) = 0
> >
> > (c) Pr(I live) = 0, Pr(I die) = 1

_________________________________________________________________
Buy now @ Tradingpost.com.au
http://a.ninemsn.com.au/b.aspx?URL=http%3A%2F%2Fad%2Eau%2Edoubleclick%2Enet%2Fclk%3B23850242%3B12217581%3Bw%3Fhttp%3A%2F%2Fwww%2Etradingpost%2Ecom%2Eau%2F%3Freferrer%3DnmsnHMetag&_t=11482&_r=emaildec05&_m=EXT

Saibal Mitra

unread,
Dec 3, 2005, 9:16:17 AM12/3/05
to Brent Meeker, everything

> But you're assuming laws of physics and a physical basis for
consciousness. I
> thought the idea was to take conscious moments as basic. I'm fine with
taking
> physics as basic - but then what's the point of talking about observer
moments;
> conscious observations are then some kind of emergent phenomena and
they're
> connected by physical causation.
>


Yes, but it's a fact that there exists laws of physics. I am of the opinion
that what really exists is an ensemble of algorithms and that the laws of
physics is a consequence of this. Whatever your starting point, you'll end
up with an absolute measure over the set of all OMs.

Saibal Mitra

unread,
Dec 3, 2005, 9:27:45 AM12/3/05
to Stathis Papaioannou, everyth...@eskimo.com, Quentin Anciaux, George Levy
This doubling of the absolute measure is important. In another posting you
wrote about being teleported to many places and then being annihilated
everywhere except at the original place. This won't affect the probability
of being alive at the original place. But in a QC experiment where you have
many outcomes, all leading to death except one, the probability of
experiencing that branch is very small.


----- Original Message -----
From: "Stathis Papaioannou" <stathispa...@hotmail.com>
To: <smi...@zeelandnet.nl>; <everyth...@eskimo.com>
Sent: Saturday, December 03, 2005 11:38 AM
Subject: Re: Quantum Immortality and Information Flow

Stathis Papaioannou

unread,
Dec 3, 2005, 11:53:22 PM12/3/05
to smi...@zeelandnet.nl, everyth...@eskimo.com, quentin...@advalvas.be, gl...@quantics.net

There is, of course, a difference between being duplicated so that there are
multiple copies of you in the one Universe, as in teleportation, and being
duplicated along with the rest of the Universe as a result of MWI branching.
In the former case your relative measure increases and problems will arise
when it comes to deciding who will get the spouse, house, bank account etc.
In the latter case your relative measure stays the same because everything
else is duplicated along with you and nothing will seem to have changed. You
agree that in the teleportation example if your duplicate is instantaneously
annihilated the moment he comes into being, you will continue living with
probability 1, as if the duplication had not taken place. On the other hand,
in the MWI branching example, you would argue that if your duplicate in one
of the branches is annihilated, then your subjective probability of survival
is 1/2.

Now, suppose that instead of just you the entire Earth, or Galaxy, or
Universe is duplicated along with you, while as before your duplicate (and
only he) is annihilated the moment he comes into being on the new Earth (or
Galaxy, or Universe). It could be argued that your measure relative to the
rest of the Universe (or that part of it which is duplicated) has now
decreased. Is your expectation of survival in this case more like the
original teleportation example, or more like the MWI branching example?

Stathis Papaioannou

Saibal Mitra writes:

_________________________________________________________________
Express yourself instantly with MSN Messenger! Download today - it's FREE!
http://messenger.msn.click-url.com/go/onm00200471ave/direct/01/

Saibal Mitra

unread,
Dec 4, 2005, 8:54:34 PM12/4/05
to Stathis Papaioannou, everyth...@eskimo.com, quentin...@advalvas.be, gl...@quantics.net
I still think that if you double everything and then annihilate only the
doubled person, the probability will be 1. This is simply a consequence of
using the absolute measure. The idea is that the future is ''already out
there''. So, the correct picture is not that suddenly the plenitude is made
larger because a copy of the person plus (part of) his universe is appended
to the plenitude. The plenitude itself is a timeless entity, containing all
possible states. If someone wants to carry out a duplication experiment then
the results of that are ''already'' present in the plenitude.

When death can be ignored then the apparent time evolution can be described
by a relative measure which is given as the ratio of absolute measures taken
before and after an experiment (as pointed out by George Levy in a previous
reply). Note that the locality of the laws of physics imply that you can
never directly experience the past. So, if you measure the z-component of a
spin polarized in the x-direction, you will find yourself in a state where
you have measured, say, spin up, while you have a memory of how you
prepaired the spin of the particle, some time before you made the
measurement. One thus has to distinguish between the three states:

S1: the experimenter prepaires the spin of the particle

S2: the experimenter finds spin up while having the memory of being in S1

S3: the experimenter finds spin down while having the memory of being in S1

These three states are ''timeless'' elements of the plenitude. They have
their own intrinsic measures. I challenge people on this list to explain why
this is not the case. If you have a plenitude you have everything. So, S1,
S2 and S3 are just ''out there''. The measure of S2 and S3 are half that of
S1. The probability of being in either S2 or S3 is thus the same as being in
S1. But if measuring spin down leads to instant death, then the probability
of being alive after the experiment is half that of being alive before the
experiment.

Stathis Papaioannou

unread,
Dec 5, 2005, 1:47:03 AM12/5/05
to smi...@zeelandnet.nl, everyth...@eskimo.com, quentin...@advalvas.be, gl...@quantics.net

I'm perhaps missing something here. In a no-collapse interpretation of QM,
doesn't "everything double" every moment? So, if only one of the doubled
versions of a person is annihilated, doesn't this mean the probability of
survival is 1?

Although the plenitude is timeless, containing all possible states, we
self-aware substructures certainly experience the illusion (if you prefer,
the emergent phenomenon) of time. When I consider which parts of the
plenitude are of selfish interest to me, I need only consider those parts
which I perceive to be in my unique present or in my pluripotent future.
More narrowly, I need only consider those parts which I perceive to be in my
immediate future - my next conscious moment - since it is only through this
process, a moment at a time, that potential future experiences become actual
present experiences, rather than irrelevant side-branches, such as the
version of me who migrated to New Zealand when I was 5 years old. What this
means is that when I consider the subjective probability of what will happen
to me in the next moment, I don't have to think about those versions of me
which are in the past, in the far future, have turned into George Bush or
are dead. I could put this differently: as a matter of fact, it is not
incorrect to say that I will suddenly become 5 years old again, or turn into
Geoge Bush, since all these states exist timelessly in the plenitude and
there is no absolute sense in which it can be said that one state "becomes"
another state. However, from my selfish point of view, when I consider the
next moment, all those other states are irrelevant. The only relevant states
are those which count as my "next moment", as normally understood by humans.
Where there are multiple candidate "next moments", the probability that I
will experience one of them depends on the relative measure of each in the
plenitude. If there is no candidate "next moments" at all, then I will die.

Stathis Papaioannou

Saibal Mitra writes:

_________________________________________________________________

Bruno Marchal

unread,
Dec 5, 2005, 11:22:26 AM12/5/05
to Russell Standish, Fabric-o...@yahoogroups.com, everyth...@eskimo.com

Le 03-déc.-05, à 11:06, Russell Standish a écrit :

> On Mon, Nov 21, 2005 at 03:39:58PM +0100, Bruno Marchal wrote:
>> Observation is implicitly defined here by measurement capable of
>> selecting alternatives on which we are able to bet (or to gamble ?).
>> The french word is "parier".
>>
>
> Well at least this isn't a problem of translation. But I still have
> difficulty in understanding why Pp=Bp & -B-p should be translated into
> English as "to bet on p" (or for that matter pourquoi on devrait
> le traduire par "a parier a p")
>
> For me Bp & -B-p is simply a statement of consistency - perhaps what
> we mean by mathematical truth.


~Bf, which is equivalent to D~f, or Dt can be considered as a
consistency statement in case "B" represents some "provability" notion.
Indeed ~Bf = NOT PROVABLE FALSE, and by definition a machine is
consistent if the machine does not prove the false.
And when we will "interview" the Lobian machine, "B" will indeed denote
some provability-by-the lobian-machine notion.

But here we were in a somehow more abstract (thus more easy!)
presentation, which at this stage let completely open how "B" will be
interpreted. In that case you can also consider the formula ~Bf, or Dt,
as a consistency statement, just a more abstract one.

Now in term of a Kripke frame/multiverse: Dt means "I am alive", or "I
am in a transitory state", or "I have access to at least one accessible
world", etc.

More generally ~Bp (or D~p) is a stronger "consistency statement"
meaning that I cannot prove p, meaning that there is an accessible
world where ~p is true.

Now, Bp & ~B~p, that is Bp & Dp, is a much stronger statement saying
that not only p is consistent or possible, but that p is also
"provable/necessary/", which in multiverse term, means that p is true
in all accessible worlds.

So Bp means (in some world alpha) "p is true in all accessible (from
alpha) worlds". Note that if B represents some provability predicate
written in first order logic, then by the most fundamental COMPLETENESS
theorem of Godel (1930, one year before his incompleteness result) it
can be shown that Bp is true if and only if p is true in all the model
of the theory/machine. So Bp is *the* natural candidate for asserting
that "p has probability one", given that Bp means "p is true in all
accessible world".

But now, by the second incompleteness theorem, the machine cannot prove
that Bp -> Dp, because that would imply Bt -> Dt, and, giving that Bt
is provable, this would entail Dt is provable, but for sound lobian
machine Dt -> ~BDt, that is "if I am consistent then I cannot prove my
consistency".
In term of (arbitrary) multiverse, it is even simpler: we just could be
in a cul-de-sac world, where Bf is always true, and Dt is always false,
and clearly this shows that Bp cannot, in general, be taken for
"probability of p is equal to 1": we need to add explicitly the
assumption that there is at least one accessible world!

So "probability of p (in world alpha) is equal to one" is well captured
by Bp&Dp (in world alpha). This means (Kripke-semantically) "p is true
in all accessible world & there is at least one possible world where
true is false".

Of course G* knows that Bp is actually equivalent with Bp & Dp, but the
machine has no way to know that, so, from the machine's point of view,
the logic of the new box B'p defined by Bp & Dp, will be a different
logic. Exercise: show that B'p -> D'p.

And then, if p is verifiable or just attainable by the universal
dovetailer, then it can be shown that p obeys to p->Bp, and this leads
B'p to a quantum logic. The" probability 1" pertaining to the
"provable-and-consistent" verifiable (DU-accessible) proposition gives
a non boolean quantum logic.

Tell me if this is clear enough. Euh I hope you agree that "To bet on
p" can be used for the probability one, of course. If that is the
problem, remember I limit myself to the study of the "probability one"
and its modal dual "probability different from zero".

I must go now and I have not really the time to reread myself, hope I
manage the "s" correctly. Apology if not. Please ask any question if I
have been unclear.

Bruno


http://iridia.ulb.ac.be/~marchal/


Russell Standish

unread,
Dec 5, 2005, 5:46:53 PM12/5/05
to Bruno Marchal, Fabric-o...@yahoogroups.com, everyth...@eskimo.com
On Mon, Dec 05, 2005 at 03:58:20PM +0100, Bruno Marchal wrote:
> >
> >Well at least this isn't a problem of translation. But I still have
> >difficulty in understanding why Pp=Bp & -B-p should be translated into
> >English as "to bet on p" (or for that matter pourquoi on devrait
> >le traduire par "a parier a p")
> >
> >For me Bp & -B-p is simply a statement of consistency - perhaps what
> >we mean by mathematical truth.
>

...

> So "probability of p (in world alpha) is equal to one" is well captured
> by Bp&Dp (in world alpha). This means (Kripke-semantically) "p is true
> in all accessible world & there is at least one possible world where
> true is false".

...


>
> Tell me if this is clear enough. Euh I hope you agree that "To bet on
> p" can be used for the probability one, of course. If that is the
> problem, remember I limit myself to the study of the "probability one"
> and its modal dual "probability different from zero".
>
> I must go now and I have not really the time to reread myself, hope I
> manage the "s" correctly. Apology if not. Please ask any question if I
> have been unclear.
>
> Bruno
>
>
> http://iridia.ulb.ac.be/~marchal/

Yes - this does make sense. Kripke frames are a good way of explaining
why Bp&Dp captures prob=1 type statements. I'm still not sure "bet
on" is the correct verb though, as in normal life one bets on things with
prob <1 (eg on a horse winning a race). Prob=1 is a "sure bet", but I
can't quite think of an appropriate verb.

Bruno Marchal

unread,
Dec 6, 2005, 11:46:52 AM12/6/05
to Russell Standish, Fabric-o...@yahoogroups.com, everyth...@eskimo.com

Le 05-déc.-05, à 22:49, Russell Standish a écrit :


Well thanks, and "sure bet" is probably better than my phrasing. It is
really the particular case of "probability one". I will surely explain
asap why Quantum Logic can be interpreted as the logic of "probability
one" in quantum mechanics. This has been single out by Maria Louisa
Dalla Chiara, the quantum logician of Florence (Firenze) Italy, but it
was the main basic motivation of von Neumann when he opened the field
of quantum logic.

Let me give you the main result (by Goldblatt) connecting quantum logic
and modal logic. It is a theorem of representation of quantum logic
into modal logic. There exist a function R translating quantum logic in
modal logic. R can be described recursively in the following way:

The atomic statement p are interpreted by the "quantization(*)" BDp.
i.e R(p) = BDp
the negative statement ~A is interpreted by B~R(A), i.e, R(~A) = B~R(A)
conjonction: R(A & B) = R(A) & R(B).

Goldblatt proved that MQL proves A iff the modal logic B proves R(A)
MQL is for Minimal Quantum Logic (the result can be extended to the
standard orthomodular quantum logic).

And B is the logic having as 1) axioms: K, T, and LASE (p -> BDp), the
"little abstract Schroedinger equation); 2) inferences rules: Modus
ponens and Necessitation rule.
(Those who have forget should search LASE in the archive).

Exercise: could someone guess which multiverses (W,R) makes LASE valid
(true in all worlds for all "illumination" on the worlds)? Answer: R
need just to be symmetrical. See why?

More explanation soon. I intend to answer an off-line mail by Stathis
who asks good questions, asap (not today, probably tomorrow). Later I
will explain how the translation of the UDA (Universal Dovetailer
Argument) in the language of a lobian machine gives rise to LASE for
the probability 1, the sure bets.

Cheers,

Bruno

(*) The term quantization in this setting has been introduced by
Rawling and Selesnick in a paper where they modelize a quantum NOT with
the modal logic B. Reference and abstract here:
http://portal.acm.org/citation.cfm?id=347481.


http://iridia.ulb.ac.be/~marchal/


Bruno Marchal

unread,
Dec 7, 2005, 7:55:28 AM12/7/05
to Everything-List List

Le 05-déc.-05, à 02:46, Saibal Mitra a écrit :


> I still think that if you double everything and then annihilate only
> the
> doubled person, the probability will be 1.

Actually I agree with this.


> This is simply a consequence of
> using the absolute measure.

Ah ? I am not sure this makes sense. If this makes sense, then the
Absolute Measurer and the Relative one are closer than I was used to
think.


> The idea is that the future is ''already out
> there''.

Again I agree, but I would say the 2^aleph_0 futures are "already out
there". That's why we need a measure. I could say that the
mathematical shape of that measure should be absolute (the same in all
the worlds of the multiverse). The value of the measure with respect to
the choice of some experiment is relative. Would you agree?

> So, the correct picture is not that suddenly the plenitude is made
> larger because a copy of the person plus (part of) his universe is
> appended
> to the plenitude. The plenitude itself is a timeless entity,
> containing all
> possible states. If someone wants to carry out a duplication
> experiment then
> the results of that are ''already'' present in the plenitude.


I agree if you are talking about the 3-plenitude. For the 1-plenitude,
the question is more delicate.
(G* can show that the 1 and 3 notions of plenitude are the same, but
from the machine point of view (either 1 or 3 view) this in not the
case at all: the 1-plenitude will look much vaster than the
3-plenitude. This is akin to the Skolem paradox in axiomatic set
theory, but also to some carrolian or monthy-python like fantasies
where some place look tiny as seen from outside and very big from
inside :)


>
> When death can be ignored then the apparent time evolution can be
> described
> by a relative measure which is given as the ratio of absolute measures
> taken
> before and after an experiment (as pointed out by George Levy in a
> previous
> reply).


Yes but as far as I remember older posts by George Levy, we need also
to take into account some fusion of histories, by amnesy or "quantum"
erasure, and this prohibits trust in the use of intuitive
probabilities. Then the interview of the Universal machine explains
somehow why things are counter-intuitive there (self-reference
limitations).


> Note that the locality of the laws of physics imply that you can
> never directly experience the past.


Yes but then you should make clear if you assume the laws of physics
just for illustration or as a fundamental hypothesis. From you other
recent post I guess you don't assume the physical laws, just the
algorithm (and I add the "mathematical execution of those algorithm in
platonia. OK?

> So, if you measure the z-component of a
> spin polarized in the x-direction, you will find yourself in a state
> where
> you have measured, say, spin up, while you have a memory of how you
> prepaired the spin of the particle, some time before you made the
> measurement. One thus has to distinguish between the three states:
>
> S1: the experimenter prepaires the spin of the particle
>
> S2: the experimenter finds spin up while having the memory of being in
> S1
>
> S3: the experimenter finds spin down while having the memory of being
> in S1
>
> These three states are ''timeless'' elements of the plenitude. They
> have
> their own intrinsic measures. I challenge people on this list to
> explain why
> this is not the case. If you have a plenitude you have everything. So,
> S1,
> S2 and S3 are just ''out there''.

OK.


> The measure of S2 and S3 are half that of
> S1. The probability of being in either S2 or S3 is thus the same as
> being in
> S1.


OK (relatively). 3-point-of- view talk.

> But if measuring spin down leads to instant death, then the probability
> of being alive after the experiment is half that of being alive before
> the
> experiment.


Except that "death" has no 1-meaning, and should not be taken into
account for evaluating a probability question. Here too George Levy
argued some time ago that, strictly speaking the probability to find
oneself 1-alive is always 1. But here too it is a little delicate
because it is a typical "pure theological truth", it belongs to G* \ G.

I recall G formalizes correctly and completely what sound machines can
prove about themselves, and that G* formalizes correctly and completely
what is true about the sound machine, but not necessarily provable
(that's mainly Solovay theorem). G* \ G literally axiomatizes what
sound machines can "correctly hope" about themselves.

Your post makes me doubt the difference between Absolutist and
Relativist, about measure, is less big than I was used to think.

Bruno

http://iridia.ulb.ac.be/~marchal/

Bruno Marchal

unread,
Dec 7, 2005, 8:29:29 AM12/7/05
to Everything-List List

Le 03-déc.-05, à 11:12, Stathis Papaioannou a écrit :


No causal link. OK. But there is an arithmetical or
computer-science-theoretical link. This includes memories, consistency
conditions, etc.

>
> I am agnostic regarding the question of whether OMs are primitive or
> derivative. The world could be as it appears: the physical universe
> (whatever that means) gives rise to certain special physical processes
> which result in moments of conscious experience, and those moments
> which are related through being the product of circumscribed subsets
> of physical processes constitute a stream of consciousness in an
> individual life. On the other hand, in a world where exactly the same
> OMs as postulated in the previous sentence exist, but all mixed up and
> not connected to any (or any particular) physical process, exactly the
> same individual streams of consciousness would result.


OK. So why ever postulate a physical world, given that the OMs you
describe already exist independently of us (in arithmetical platonia),
and that nobody has ever succeed in explaining what a primitive
physical world could be, and that nobody has ever succeed in relating
OMs as lived by people and some putative concrete substantial reality?
I think that with OCCAM, any notion of computationalist OMs, makes the
idea of a ontologically primitive physical multiverse useless. (and the
Olympia/movie-graph makes it senseless, but here I would say that
remark is off-topic).

Bruno

http://iridia.ulb.ac.be/~marchal/


George Levy

unread,
Dec 8, 2005, 4:49:32 PM12/8/05
to Everything List
Bruno Marchal wrote:

Le 05-déc.-05, à 02:46, Saibal Mitra a écrit :


I still think that if you double everything and then annihilate only the
doubled person, the probability will be 1.



Actually I agree with this.



So far we have been talking about splitting universes and people. Let's consider the case where two branches of the universe merge. In other words, two different paths eventually happen to become identical - Of course when this happens all their branching futures also become identical. Would you say that such a double branch has double the measure of a single branch even though the two branches are totally indistinguishable? How can you possibly assert that any branch is single, double, or a bundle composed of any number of identical individual branches?

George

Quentin Anciaux

unread,
Dec 8, 2005, 6:25:32 PM12/8/05
to everyth...@eskimo.com
Hi Georges,

if you start from OMs as basic, then a branch is a set of OMs (only
"consistent"/ordered set ?). Then it means a branch is unique. Some part of
different branches could overlap, but as I don't understand what could be an
absolute measure (meaning it never change and is fixed forever) between all
branches, I don't see how to assert the measure of a branch... Also viewing
from this point each 1st pov "lives" in its own branch (as a branch is an
ordered set of OMs which in turn is associated to a 1st person).

Quentin

Stathis Papaioannou

unread,
Dec 9, 2005, 7:58:13 AM12/9/05
to gl...@quantics.net, everyth...@eskimo.com
George Levy writes:

>So far we have been talking about splitting universes and people. Let's
>consider the case where two branches of the universe merge. In other words,
>two different paths eventually happen to become identical - Of course when
>this happens all their branching futures also become identical. Would you
>say that such a double branch has double the measure of a single branch
>even though the two branches are totally indistinguishable? How can you
>possibly assert that any branch is single, double, or a bundle composed of
>any number of identical individual branches?

If two separate branches in the multiverse happen to become identical, so
that they can be said to fuse at this point, is it necessarily true that
their futures will also be identical from then on? For example, if two
branches are identical at a certain point, but in one branch the
gravitational constant is rising while in the other it is falling, won't
this make them split up again soon after fusing?

Stathis Papaioannou

_________________________________________________________________
realestate.com.au: the biggest address in property
http://ninemsn.realestate.com.au

Bruno Marchal

unread,
Dec 9, 2005, 1:55:21 PM12/9/05
to George Levy, Everything List

Le 08-déc.-05, à 22:21, George Levy a écrit :


> Bruno Marchal wrote:
>>
>> Le 05-déc.-05, à 02:46, Saibal Mitra a écrit :
>>
>>
>>> I still think that if you double everything and then annihilate only
>>> the
>>> doubled person, the probability will be 1.
>>
>>
>>
>> Actually I agree with this.
>>
>>
>
> So far we have been talking about splitting universes and people.
> Let's consider the case where two branches of the universe merge.

Of course this is the an hard and interesting question ... I would say
that Everett, Deutsch, Hartle somehow answer it in the quantum realm. I
would say that empirically or "apparently", at the bottom there is
neither elimination of information, nor duplication of information.
Irreversibility and non cloning.
I believe comp entails this too. Got evidence from the interview with
the Lobian Machine, but also from some intuitive way to put (first
person) measure on the computational histories generated by the UD.

> In other words, two different paths eventually happen to become
> identical -

At the bottom I don't think this can happens. Like Deutch I think that
both bifurcation and fusion are really differentiation and
dedifferentiation by *apparent* lack of memory.

Remember Y = II If you "bifurcate" I think you just grow
the measure on your past. If you fuse consistently you don't change the
measure. To be sure I have also different arguments in favor of an
increase of measure when you fuse (loosing memory makes greater your
possible histories, like substracting equations in a system of
equations augments the possible number of solutions (the Galois
connection).
All this is very difficult, that I think we should take benefit of
Godel, Lob, Solovay and the discovery of the (modal) logic of
self-reference G and G* to ask the opinion of a universal machine ...

> Of course when this happens all their branching futures also become
> identical.

This is not so obvious. You should define a notion of identity for the
branches, path, etc.

> Would you say that such a double branch has double the measure of a
> single branch even though the two branches are totally
> indistinguishable? How can you possibly assert that any branch is
> single, double, or a bundle composed of any number of identical
> individual branches?

Indeed, how? And from which point of view?


Bruno

http://iridia.ulb.ac.be/~marchal/

George Levy

unread,
Dec 9, 2005, 5:08:37 PM12/9/05
to Everything List
Hi Quentin, Stathis, Bruno


Quentin Anciaux wrote:
Hi Georges,

if you start from OMs as basic, then a branch is a set of  OMs (only 
"consistent"/ordered set ?). Then it means a branch is unique. Some part of 
different branches could overlap, but as I don't understand what could be an 
absolute measure (meaning it never change and is fixed forever) between all 
branches, I don't see how to assert the measure of a branch... Also viewing 
from this point each 1st pov "lives" in its own branch (as a branch is an 
ordered set of OMs which in turn is associated to a 1st person).

  
Hi Quentin, Stathis, Bruno

It all depends how you see the plenitude, OMs and the branching. Is consciousness like a traveller in a network of roads traversing the plenitude, some roads branching some roads merging?

If yes then you could have several independent consciousness occupying the same spot, or the same OM. Then their measure at that spot is their sum. This approach is a third person point of view and it leads to the concept of absolute measure.

If you see consiousness as the road itself, then measure is not increased after a merge and does not decrease after a split. An OM is just a point on the road. If the road turns unexpectedly to avoids an obstacle (like quantum suicide or just plain death), then consiousness will just move on into a direction which has a low 3-rd person probability but unity first person probability. Viewing consciousness as a network of roads is a first person point of view and it leads to the concept of relative measure: Measure is always 1 where you are. >From a given point you may reach many points - Then measure increases with respect to that point. Or reversibly, from many points you may reach only one point. Then measure decreases.


Bruno writes:

        >neither elimination of information, nor duplication of information.

The crux of the matter is the concept of indistinguishability: whether you consider two identical persons (OMs) occupying two identical universes the same person (point on the road). It is clear that if you consider the problem from the information angle, then duplication of information does not increase the measure of that information. This would support the relative interpretation of measure.

George

Stathis Papaioannou

unread,
Dec 10, 2005, 8:24:23 AM12/10/05
to gl...@quantics.net, everyth...@eskimo.com
George Levy writes:


In addition to the above arguments, consider the problem from the point of
view of the subject. If multiple copies of a person are created and run in
parallel for a period, what difference does this make to his experience? It
seems to me that there is no test or experiment the person could do which
would allow him to determine if he is living in a period of high measure or
low measure. If an OM is the smallest discernible unit of conscious
experience, it therefore seems reasonable to treat multiple instantiations
of the same OM as one OM.


Stathis Papaioannou

Bruno Marchal

unread,
Dec 10, 2005, 10:32:16 AM12/10/05
to Stathis Papaioannou, Everything-List List

Le 10-déc.-05, à 13:24, Stathis Papaioannou a écrit :

> In addition to the above arguments, consider the problem from the
> point of view of the subject. If multiple copies of a person are
> created and run in parallel for a period, what difference does this
> make to his experience? It seems to me that there is no test or
> experiment the person could do which would allow him to determine if
> he is living in a period of high measure or low measure.


To determine this with certainty? I agree with you in that case. But we
can make "sure bets".

Take the iterated self-duplication (thought) experiment: You are
"read", "cut" and then "pasted" in two identical rooms except one has
1 drawn on the wall where the other has 0.
Then "each of you" do it again and again.
After 64 duplications you stop. A vast majority among the 2^64 "yous"
will confirms they bet on their normality. Normal experience here is
guarantied by the incompressible information of most bits sequences
(provable by a simple combinatorial analysis).
This is equivalent of betting the halving of the intensity of a beam
of x polarized photons going through a y analyser, with Everett QM.

> If an OM is the smallest discernible unit of conscious experience, it
> therefore seems reasonable to treat multiple instantiations of the
> same OM as one OM.


OK but with comp I have argued that OMs are not primitive but are
"generated", in platonia, by the Universal Dovetailer. A 3- OM is just
an UD-accessible state, and the 1-OMs inherit relative probabilities
from the computer science theoretical structuring of the 3-OMs.

It is the 1 3 person distinction which forces, I think, the relativity
or conditionality of the measure. There is no a priori means to know if
we are, just now, in a Harry Potter (abnormally informative) type of
OM, but we can always bet our next OMs will belong to the set of their
most normal continuators (probably the product of long (deep)
computations with stability on dovetailing on the reals or noise).

Bruno

http://iridia.ulb.ac.be/~marchal/


Bruno Marchal

unread,
Dec 10, 2005, 10:43:24 AM12/10/05
to George Levy, Everything List

Le 09-déc.-05, à 22:44, George Levy a écrit :

> The crux of the matter is the concept of indistinguishability:
> whether you consider two identical persons (OMs) occupying two
> identical universes the same person (point on the road). It is clear
> that if you consider the problem from the information angle, then
> duplication of information does not increase the measure of that
> information. This would support the relative interpretation of
> measure.

Yes. And this already comes from the fact that the
"indistinguishabilitty/distinguishabilitty" crux is itself relative. By
loosing memory something distinguishable can become indistinguishable,
augmenting the class of (normal) self-consistent extensions.

Then I am open that from the 1 point of view, fusion increases measure,
duplication decreases measure; although from the 3 pov it is the
contrary.

Bruno

http://iridia.ulb.ac.be/~marchal/

Stathis Papaioannou

unread,
Dec 11, 2005, 6:58:27 AM12/11/05
to mar...@ulb.ac.be, everyth...@eskimo.com
Bruno Marchal writes:

>Le 10-déc.-05, à 13:24, Stathis Papaioannou a écrit :
>
>>In addition to the above arguments, consider the problem from the point of
>>view of the subject. If multiple copies of a person are created and run in
>>parallel for a period, what difference does this make to his experience?
>>It seems to me that there is no test or experiment the person could do
>>which would allow him to determine if he is living in a period of high
>>measure or low measure.
>
>
>To determine this with certainty? I agree with you in that case. But we can
>make "sure bets".
>
>Take the iterated self-duplication (thought) experiment: You are "read",
>"cut" and then "pasted" in two identical rooms except one has 1 drawn on
>the wall where the other has 0.
>Then "each of you" do it again and again.
>After 64 duplications you stop. A vast majority among the 2^64 "yous" will
>confirms they bet on their normality. Normal experience here is guarantied
>by the incompressible information of most bits sequences (provable by a
>simple combinatorial analysis).
>This is equivalent of betting the halving of the intensity of a beam of x
>polarized photons going through a y analyser, with Everett QM.

What I meant above was that the presence of parallel copies per se cannot
directly change the quality of the first person experience of any of the
copies. It may be possible to infer the presence of other copies by indirect
means; for example, in a closed system a high measure period may be
characterised by faster oxygen consumption.

I'm not sure what you mean by "[a] vast majority among the 2^64 "yous" will
confirms they bet on their normality", but I'm guessing that you are
referring to the idea that if you bet on being sampled from high measure
group rather than the low measure group, you are more likely to be right.
This method has its problems. Consider this thought experiment which I
proposed a few months ago:

You find yourself alone in a room with a light that alternates red/green
with a period of one minute. A letter in the room informs you that every
other minute, 10^100 copies of you are created and run in parallel for one
minute, then shut down. The transition between the two states (low measure/
high measure) corresponds with the change in the colour of the light, and
you task is to guess which colour corresponds to which state.

The problem is, whether the light is red or green, you could argue that you
are vastly more likely to be sampled from the 10^100 group. You might decide
to say that *both* red and green correspond to the larger group, because if
you say this 10^100 copies in the multiverse will be correct and only one
copy will be wrong. But clearly, this tyranny of the majority strategy
brings you no closer to the truth. If you tossed a coin, at least you would
have a 1/2 chance of being right.

>>If an OM is the smallest discernible unit of conscious experience, it
>>therefore seems reasonable to treat multiple instantiations of the same OM
>>as one OM.
>
>
>OK but with comp I have argued that OMs are not primitive but are
>"generated", in platonia, by the Universal Dovetailer. A 3- OM is just an
>UD-accessible state, and the 1-OMs inherit relative probabilities from the
>computer science theoretical structuring of the 3-OMs.
>
>It is the 1 3 person distinction which forces, I think, the relativity or
>conditionality of the measure. There is no a priori means to know if we
>are, just now, in a Harry Potter (abnormally informative) type of OM, but
>we can always bet our next OMs will belong to the set of their most normal
>continuators (probably the product of long (deep) computations with
>stability on dovetailing on the reals or noise).

Are OMs directly generated by the UD, or does the UD generate the physical
(apparently) universe, which leads to the evolution of conscious beings, who
then give rise to OMs?

Stathis Papaioannou

_________________________________________________________________
Win over $10,000 in Dell prizes this Christmas
http://ninemsn.com.au/share/redir/adTrack.asp?mode=click&clientID=151&referral=Hotmailtagline&URL=http://shoppingau.ninemsn.com.au/compintro.aspx?compid=174

Bruno Marchal

unread,
Dec 12, 2005, 12:14:43 PM12/12/05
to Stathis Papaioannou, everyth...@eskimo.com

Le 11-déc.-05, à 11:58, Stathis Papaioannou a écrit :

> You find yourself alone in a room with a light that alternates
> red/green with a period of one minute. A letter in the room informs
> you that every other minute, 10^100 copies of you are created and run
> in parallel for one minute, then shut down. The transition between the
> two states (low measure/ high measure) corresponds with the change in
> the colour of the light, and you task is to guess which colour
> corresponds to which state.
>
> The problem is, whether the light is red or green, you could argue
> that you are vastly more likely to be sampled from the 10^100 group.
> You might decide to say that *both* red and green correspond to the
> larger group, because if you say this 10^100 copies in the multiverse
> will be correct and only one copy will be wrong. But clearly, this
> tyranny of the majority strategy brings you no closer to the truth. If
> you tossed a coin, at least you would have a 1/2 chance of being
> right.


Yes but this is due to the "shut down". (if I got correctly your
experiment).The probabilities can be taken only on the stories without
dead-ends, and I guess you consider the shut down as sort of "absolute
annihilation".
I know this is hard to believe, but apparently we are "conscious" only
because we belong to a continuum of infinite never ending stories ...

I don't believe this, but then that's what the lobian machine's
"guardian angel" G* says about that: true and strictly unbelievable.

Do you accept that your argument won't go through if the shut down are
deleted?

Bruno

http://iridia.ulb.ac.be/~marchal/


daddy...@aol.com

unread,
Dec 12, 2005, 1:18:05 PM12/12/05
to mar...@ulb.ac.be, stathispa...@hotmail.com, everyth...@eskimo.com
Bruno wrote:

In response to Stathis' thought experiment, to speak of an experiment
being "set up" in a certain way is to base probabilities on an
"irrelevant" subset of the whole, at least if the multiverse hypothesis
is true. In the Plenitude, there are an additional 10^100 copies still
existing, when you say that 10^100 copies are being shut-down. Talking
about these additional 10^100 copies is just as consistent as talking
about the original 10^100 copies (even more consistent if you consider
Bruno's statement about cul-de-sacs.

In the Plenitude, everything washes out to zero. And Bruno, I would
even say that all consistent histories wash out to zero.
Bruno, I've been following your posts about Kripke semantics and have
done the exercises, including the one about showing that you need a
symmetrical accessibility relation to have LASE. However, my initial
reaction still is that choosing a particular modal logic is scary to
me, sending up red flags about hidden assumptions that are being made
in the process. But I will continue to follow you as you present your
case.

Earlier Stathis wrote:
>> Bruno: OK but with comp I have argued that OMs are not primitive but

are "generated", in platonia, by the Universal Dovetailer. A 3- OM is
just an UD-accessible state, and the 1-OMs inherit relative
probabilities from the computer science theoretical structuring of the
3-OMs.
>

> Are OMs directly generated by the UD, or does the UD generate the
physical (apparently) universe, which leads to the evolution of
conscious beings, who then give rise to OMs?
>
> Stathis Papaioannou

It's interesting that symmetry (Bruno's requirement for LASE) has come
up lately, because Stathis' question seems to be what we are all
wondering. That's the bottom line of multiverse theories: Where does
the symmetry breaking come from? I maintain still that it can't come
from the multiverse itself. Even considering only consistent
histories, there is no asymmetry to be found. I maintain that it needs
to come from outside the multiverse, which is something that we cannot
explain.

Tom Caylor


George Levy

unread,
Dec 12, 2005, 2:34:11 PM12/12/05
to Everything List


Stathis Papaioannou wrote:

In addition to the above arguments, consider the problem from the point of view of the subject. If multiple copies of a person are created and run in parallel for a period, what difference does this make to his experience? It seems to me that there is no test or experiment the person could do which would allow him to determine if he is living in a period of high measure or low measure. If an OM is the smallest discernible unit of conscious experience, it therefore seems reasonable to treat multiple instantiations of the same OM as one OM.

Yes Stathis, I agree with you completely.



Bruno wrote:
And this already comes from the fact that the "indistinguishabilitty/distinguishabilitty" crux is itself relative. By loosing memory something distinguishable can become indistinguishable, augmenting the class of (normal) self-consistent extensions.

Bruno, I find this question extremely difficult. Is indistinguishability established at the physical level or at the psychological level? If we say it is established at the psychological level, then even mental errors ( ie.6+7=11) count in defining a whole world. This is the ultimate in relativism. I can find reasons to go either way. (Ultimately Undecided?)


Then I am open that from the 1 point of view, fusion increases measure, duplication decreases measure; although from the 3 pov it is the contrary.

I do not agree with you on this point Bruno.
>From the one person point of view measures remains constant just like the speed of light, the mass of an electron, or the number of points in a line 1 meter long or 1 kilometer long. (the number of points in a continuum is always the same no matter what the length of the line is). The one person always observes a continuum in the number of opportunities available to him no matter what his past history is.
>From the third person point of view, it makes sense to consider ratios in measures, just like it makes sense to take ratios of line segments of different lengths.

George

Stathis Papaioannou

unread,
Dec 12, 2005, 8:13:09 PM12/12/05
to mar...@ulb.ac.be, everyth...@eskimo.com
>From the third person perspective, the annihilation of the 10^100 copies
could be seen as 10^100 dead ends. (In fact, when I originally proposed this
experiment, Hal Finney thought it represented the ultimate in mass murder.)
If I were one of the 10^100, however, I wouldn't be worried in the slightest
about the prospect of dying, because as long as at least one copy survives,
this guarantees that I survive. This may go again intuition, but if you give
up the notion of an immaterial soul, there is no reason why there should be
a one to one relationship between earlier and later versions of a person.

Stathis Papaioannou

_________________________________________________________________
Is your PC infected? Get a FREE online computer virus scan from McAfee®
Security. http://clinic.mcafee.com/clinic/ibuy/campaign.asp?cid=3963

Stathis Papaioannou

unread,
Dec 12, 2005, 8:28:10 PM12/12/05
to daddy...@aol.com, mar...@ulb.ac.be, everyth...@eskimo.com
Tom Caylor writes:

>In response to Stathis' thought experiment, to speak of an experiment being
>"set up" in a certain way is to base probabilities on an "irrelevant"
>subset of the whole, at least if the multiverse hypothesis is true. In the
>Plenitude, there are an additional 10^100 copies still existing, when you
>say that 10^100 copies are being shut-down. Talking about these additional
>10^100 copies is just as consistent as talking about the original 10^100
>copies (even more consistent if you consider Bruno's statement about
>cul-de-sacs.
>
>In the Plenitude, everything washes out to zero. And Bruno, I would even
>say that all consistent histories wash out to zero.

Doesn't this ignore the concept of measure in the multiverse? If I buy a
lottery ticket there are an infinite number of versions of me who win and an
infinite number of versions who lose, but in some sense there have to be
"more" losers than winners, which is why I don't buy lottery tickets.

Bruno Marchal

unread,
Dec 13, 2005, 5:41:27 AM12/13/05
to Stathis Papaioannou, everyth...@eskimo.com

Le 13-déc.-05, à 02:07, Stathis Papaioannou a écrit :

>> From the third person perspective, the annihilation of the 10^100
>> copies
> could be seen as 10^100 dead ends. (In fact, when I originally
> proposed this experiment, Hal Finney thought it represented the
> ultimate in mass murder.) If I were one of the 10^100, however, I
> wouldn't be worried in the slightest about the prospect of dying,
> because as long as at least one copy survives, this guarantees that I
> survive. This may go again intuition, but if you give up the notion of
> an immaterial soul, there is no reason why there should be a one to
> one relationship between earlier and later versions of a person.

OK. But from this I deduce that we were agreeing. Eventually this means
we don't take the dead ends into account when computing probabilities
for future extensions of oneself.

Bruno


http://iridia.ulb.ac.be/~marchal/


Bruno Marchal

unread,
Dec 13, 2005, 6:16:07 AM12/13/05
to George Levy, Everything List

Le 12-déc.-05, à 19:37, George Levy a écrit :

>
>
> Stathis Papaioannou wrote:
>
>> In addition to the above arguments, consider the problem from the
>> point of view of the subject. If multiple copies of a person are
>> created and run in parallel for a period, what difference does this
>> make to his experience? It seems to me that there is no test or
>> experiment the person could do which would allow him to determine if
>> he is living in a period of high measure or low measure. If an OM is
>> the smallest discernible unit of conscious experience, it therefore
>> seems reasonable to treat multiple instantiations of the same OM as
>> one OM.
>
> Yes Stathis, I agree with you completely.


This is delicate and I think we should really make the disctinction
between the 3-OMs and the 1-OMs. Difference of measure of 3-OM does not
change the quality of the 1-OMs experiences, but could change the
relative probability of having some next 1-OMs.

To justify this, I point you on the UDA, or to the lobian interview.
(We are coming back on this).

>
>
> Bruno wrote:
>> And this already comes from the fact that the
>> "indistinguishabilitty/distinguishabilitty" crux is itself relative.
>> By loosing memory something distinguishable can become
>> indistinguishable, augmenting the class of (normal) self-consistent
>> extensions.
>
> Bruno, I find this question extremely difficult. Is
> indistinguishability established at the physical level or at the
> psychological level?


Psychological. Remember that with comp we take for granted some amount
of folk or Grandmother psychology (enough for saying purposefully "yes"
to the doctor). But then by the UDA, comp entails the complete
structure of the physical laws. Now the goal is to make the derivation
of the physical laws, so that we can test comp by comparing the
comp-physics with the traditional empirical physics. With such kind of
approach it is just forbidden to invoke anything physical as granted;
we can certainly not take a physical multiverse or a physically based
indistinguishability for granted.

> If we say it is established at the psychological level, then even
> mental errors ( ie.6+7=11) count in defining a whole world.

"6+7=11" is not a mental error. It is just a false proposition. I guess
you mean something like "B(6+7=11)". This is a *mental error*, where
the "mentality" has been supposed to be captured by some modal
epistemic logic "B" (a modal box). And then "6+7=11" is akin to a sort
of white rabbit or flying pig, those which, of course we need still to
justify the extreme rarity.

> This is the ultimate in relativism. I can find reasons to go either
> way. (Ultimately Undecided?)


And now this makes sense indeed and the "Ultimately Undecided" is close
to the "Forever Undecided" which will be tackle by the self-reference
logics G and G*.

>
>> Then I am open that from the 1 point of view, fusion increases
>> measure, duplication decreases measure; although from the 3 pov it is
>> the contrary.
>
> I do not agree with you on this point Bruno.
> >From the one person point of view measures remains constant just
> like the speed of light, the mass of an electron, or the number of
> points in a line 1 meter long or 1 kilometer long. (the number of
> points in a continuum is always the same no matter what the length of
> the line is). The one person always observes a continuum in the number
> of opportunities available to him no matter what his past history is.

That's true, but only for a notion of "actual 1-OMs", not necessarily
for the 3-prediction on some possible (future) 1-OMs.

> >From the third person point of view, it makes sense to consider
> ratios in measures, just like it makes sense to take ratios of line
> segments of different lengths.


OK.


Bruno


http://iridia.ulb.ac.be/~marchal/


It is loading more messages.
0 new messages